Вы находитесь на странице: 1из 46

4.

4.10

a) The system is described by

The system is described by

Since there is azimuthal symmetry, choosing the z-axis through q,


out =
out =
in =

B l r l1 Pl +

1
4 0

B l r l1 Pl +

1
4 0

A l r l Pl +

1
4 0

q
r>

q
r>

q
x |
|x

r<
r>

r<
r>

Pl
a) Since there is azimutal symmetry,

Pl

r, =

Boundary conditions: At the surface, r = d = r > , r = a = r < .

r,
= xE
= x
D

1) out = in |r=a , or
2) r in
k

D r = x lA l r l1 l + 1B l r l2 P l cos

B l = A l a 2l+1
= r out |r=a , or letting k = 0

lA l a l1 Pl +

A l r l + B l r l1 Pl cos

Also

q a
l
d
dl

l1

Pl

l + 1B l a l2 Pl +
l

l + 1A l a
l

l1

l1
q
Pl + l a l
d
d

q a
l
d
dl

l1

Pl

Pl

between the spheres,

D r dr 2 = Q,

or

A l = 0, B l = 0, l 0 D r =

a1 kl
Al =
1 + kl + 1d l+1
Bl =

and is independent of r.

Thus

D r dr 2 = 2B 0

a1 kla 2l+1
1 + kl + 1d l+1

0
1

d cos + d cos
0

Q
B0 =
2 0 1 +

4
Remember that P l = 2+1
Y 0l , and substitute the above coefficients into the expansion to get the
answer requested by the problem.

=
E

Q
2 0 1 +

xB 0
r2
= 2B 0 0 + = Q

r 2

b)

D r dA = D r A = f A f = D r = xE r
f =

Q
2 0 1 +

r 2

cos 0

ytRZ{BU F$st8$t +$st

_YkR${R x(xU 7BjZ$BtR B VRR$t8t 'E

cos < 0

_BAj8 ;d

c)

wsD qY {Ys ItR$k

pol dV = pol A = PdV = PA pol = P = 0 e E

'wnD )

Q
= x/ 0 1
21 + 0 r 2

pol

n$
$
R
&w  yD&w{BR D\&w*D L &w*  D  &w*  $D  &w*  Di
1
1
1

qY 8Zj$}Bj 8B8t

Notice
RfE

Q
= 0 E, as expected.
21 + 0 r 2

pol + f = tot =

*Z IsU !{BA dM 1((d

More Problems for Chapter 4

Q
f =
,
21 + 0 r 2


)

7$t{

hfE w?D

 ED Ryfh E w(D\d L 7E$b1  7E$  7nE$b1 i


f
f L ED

1f L d w f


I I If nI
I
VfE
w , * D 'w
DQ+ )

;$

 ED Ryf h E w(D +\d  wdDE i\d L 7E$b1 i


f
f L ED

1f L d w f
;$

f L E )BII hfE w(D %st$RYR ZtjRR f L E )%t 3ZY8B RfE %st$RYR


RfE  ABY f stI E 8ZR A BII N f ) 1i L d stI E ) 1> L dU

1f L d wf  ED E
hf w(D
Rf)1i Ld,E)1>Ld ) 1\d L wdD>Ld 7iRyf
;$ wf L ED

$R BII $P

$P

$R %t

qYPB PB tBtT%st$RY$t

 d 1wffdD

>


Ryf

 ED F QfLE

1f L d wf

qY hR bB RR BP tBtT%st$RY$t 8B8tR s

Rd,d

Rn,d

Rn,n

bwd b 7D

wd b 7D

bwd  7D

w?

Q?fLE

wf L ED

1d
dE$

 dDf /?)(

Ry

1$

nx
dE$

Ryn

Ryn

wAD qY {Ys ItR$kU

'wnD )

1$ 1

\&w  yD&w{BR   dD L &w  yD&w{BR  L dD  &wDi

qY 8Zj$}Bj 8B8t

RfE

8
)

5
)

1f L d
;$


I I If nI
I
VfE
w  , * D ' w
D Q+ )

1$

1f L d w f
;$

 ED 8 1$ 7E* Q*I\yf h E wdD L yf h E wdD  1& i


f,(
f
f
ED
I

wf L

\yf hf wdD L yf hf wdD  1&f( i&E( ) R

qY hR bB RR BP tBtT%st$RY$t 8B8tR sU

R1,(

Ry1 

1f L d
;$

R1,EW)(

\yf wd L wdDf D  1&f( i&E(

)(

5
R;,(

Ry; 

R;,EW)(

bY

)(

n(

$R $tR$I Y %BjZ8

~

qYPB Y RZPs{ $tsj %st$RYRU

wn
D )

d
;$6(

yt Y

?r

;$

1f L d

fE

1$6(

RfE

VfE w, *D
fLd

R 3 yf \d L wdDf i  1&f,(
hf w{BR D
;$6(
fLd
f)(

wID qY -s{ }Bt$sj $t Y w?

 rD }jst
w,  )

R d
1$6(

$b1D )

;$6(

d L wyb D

R y1
n

;$6(

n N on wn( D )

$1

L y1

d
1

 ;$6R

333

1R

1R

wn

&n wnI  n( D

Q+ I

n N on wnI D

N znID wnI D&n wnI  n( DQyI L


n(

P
L 333

Q+ I )

8
~

&n wnI  n( Dn N nI wnIDQ+ I

nI N \n wnI D&n wnI  n( Di  wnIDnI N \n&n wnI  n( Di

Vs$t Y RZPs{ $tsj %st$RYR R$t{

y1
n
(

n N nI &n wnI  n( D

/n  nI/

'& wnD ) n N n&n wn  n( D

8 +
L

qY tk BP Y I$}Bj $t st j{${ hjIU

}jst {BR  ) (U

$b1D ) 

;$6(

bY${Y $R Y }Bt$sj Ak st &{$% {Ys ItR$k

3 y1 >
R y1
1
h1> w{BR D )
wn {BR   dD L 333
1>Ld
;$6( n
>)d

w,  )

wn
D )

w{D qY }Bt$sj

8 +
~

n N nI &n wnI  n( D

; ~  qYPB
8 +

)
~

n N nI &n wnI  n( D

nI DQ+
w

Q+ I

nIDQ+
w

bY${Y $R Y tk BP s I$R$AZ$Bt BP {Ys ItR$k

'& wnD ) n N n&n wn  n( D

sR b$Y Y RZj BP w{D

_BAj8 ;d(

1 q
40 a

wsD qY j{${ hjIR $t Y bB $BtR 8ZR A Y Rs8 wBYb$R $ b$jj jsI B I$&t }Bt$sj I$&t{R
Abt Y $tt stI Y BY R}YR $t Y bB $BtRD V}}jk$t psZRR:R jsb $t I$j{${R Bt s psZRR$st RZPs{
BP sI$ZR

qY }Bt$sj w$t Zt$R BP wRb;$6( yDD $t

?r

}jst sR PZt{$BtR BP

by

r/a

wy c c KD

stI tB$t

n $R sjBt Y sI$sj I${$Bt Ak Rk88k


;
n N nzQy ) < p w6o L 6( o D1$1 ) <
;

qYPB Y j{${ hjI %kbY Abt Y R}Y $R

n
o

qY IBI j$t $R Y s}}B-$8s$Bt PB8

w{D stI Y RBj$I j$t $R Y -s{ {sj{Zjs$Bt BP wID

_BAj8 ;1

qY }Bt$sj s
wn
D )

<

1 $ w6 L 6( D

n
n

wAD qY P RZPs{ {Ys ItR$$R Bt Y $tt R}Y sU

IZ B s }B$t I$}Bj

n  n(
n N
;$6(
/n  n( /n
d

d
;$6(

d
;$6(

8
~

d
;$6(

nI N

8
~

n s n( U

8

d
n  nI + n nI
N n& w  n( D Q+ I ) ;$6
( ~
/n  nI /n

n&n wnI  n( D
/n  nI/

Q+ I 

d
;$6(

d
n N znI n nI
& w  n( DQyI L
;$6(
/n  nI /

/n  nI /
d

/n  nI /

nI N

nI

/n  nI/


n&n wnI  n( D Q+ I

n N nI &n wnI  n( D

Q+ I

n&n wnI  n( D Q+ I

6(
n w ) y D ) <
 ) 6( o
1$y1 6 L 6(
n w ) y D
 ) 6o

<

1$y1

Y $Bt b$YBZ Y I$j{${

6
6 L 6(

Y $Bt b$Y Y I$j{${

w{D qY }Bjs$9s$Bt $t Y $Bt b$Y Y I$j{${U

hn

) w6

 6( Don ) 1<$ 66 L 66( nn


(

qYPB Y }Bjs$9s$Bt RZPs{ {Ys ItR$k

K

< 6  6(
\hn N nzi )y ) h w ) yD )  1$y
1 6L6
(

yt Y $Bt b$YBZ Y I$j{${ Y }Bjs$9s$Bt RZPs{ {Ys ItR$k

K ) (

5.1

_BAj8 ;dn

V JZ$j$A$Z8 Y j{BRs${ PB{ Asjst{R Y s%$k 3B s h-I }Bt$sj I$&t{

~  Y j{BRs${ PB{

$R $%t Ak

B )
bY

QP
Q_

~1

Qg
Q_

The system is described by

$R Y Bsj {s}s{$st{ BP Y R{$Bt sAB% Y j$JZ$I RZPs{U

g ) g_ L gf_
<

g

_ $R Y {s}s{$st{ BP Y R{$Bt b$Y Y j$JZ$I $t Abt Y bB j{BIR stI f _ $R Y {s}s{$st{ BP


Y R{$Bt sAB% $R Y Y$Y sAB% Y j$JZ$I RZPs{ WB PB s {kj$tI${sj {s}s{$B $t %s{ZZ8 Y {s}s{$st{

} Zt$ jtY $R

g( )

$6(
KbyD

jtw

qYPB

g)

$6( wd L  D_ 1$6( wf  _D
L
jtwKbyD
jtwKbyD

d
1

B ) ~ 1

Qg
Q_

B ) B/

~
) $6jtw( Kby
D,
1

 )

$6(
\ _ L f i
KbyD 

We want to show
m =

jtw

B/ ) '$wK1  y1 D_/

wK1  y1 D'_/ jtwKbyD


6( ~ 1

0I

Suppose the observation point is moved by a displacement x, or equivlently that the loop is
displaced by x.
m , then
=
If we are to have B

m = x B
Using the law of Biot and Savart,

m =

0I
4

dl r
r3
m =

x dl

0I
4

0I
4

r dA
r2

r3
=

0I
4

x dl
r2

0I

Or,
m =

0I

5.2
If you are on the outside of the solenoid at position z 0 , by symmetry the magnetic field must be in
the z direction. Thus using the above argument, m must not depend on . Set us take far away
directed along
from the axis of the solenoid, so that we can replace the loops by elementary dipoles m
the z axis. Thus for any point z 0 we will have a contributions

a) The system is described by

r 1

m
+ m 3r 2
r 31
r2

r 1 = m
r 2 and r 1 = r 2 . Thus
where m
H=0
First consider a point at the axis of the solenoid at point z 0 . Using the results of problem 5.1,
0
NIdz
4

d m =
From the figure,
=

r r 2dA

m =

dArcos
2

= 2z

d
z
= 2
+1
3/2
2 + z 2
R 2 + z 2

R
0

0
1
NI z
+ 1z
2
z0
R 2 + z 2

Br =

0
NI z 0 + R 2 + z 20
2

dz =

0
NI z 0 + R 2 + z 20
2
z 0

z 0 + R 2 + z 20
0
NI
2
R 2 + z 20

In the limit z 0 0
Br =

0
NI
2

By symmetry, thej loops to the left of z 0 give the same contribution, so


B = B l + B r = 0 NI
H = NI
is directed along the z axis, so
By symmetry, B
=0
B
m =
is directed to the z axis. Thus for a given z, m is independent of , and consequently
if
H = NI
everywhere within the solenoid.

5.8

5.10

Using the same arguments that lead to Eq. (5.35), we can write
A =

0
4

a) From Eq. (5.35)

x cos J r ,
x |
|x

A r, =

Choose
x in the x z plane. Then we use the expansion
1
=
x |
|x

>

l,m

l
4 r < Y m , Y m , 0
l
2l + 1 r >l+1 l

The cos factor leads to only an m = 1 contribution in the expansion. Using


2l + 1 l m! P m cos
4 l + m! l

Y ml , 0 =
and

l1!
l+1!

1
ll+1

, we have on the inside

A =

0
4

>
l


1 r l P 1 cos d 3 x P l cos J r ,
l
ll + 1
r l+1

which can be written


A =

0
4

>

1
ll + 1

dr d sin cos cos r a


x |
|x

Using the expansion of 1/|x


x | given by Eq. (3.149),
1
4
=
x |
|x

0 dk coskz z

1 I k K k + cosm I k K k
<
0
>
m
<
m
>
2 0
m=1

We orient the coordinate system so = 0, and because of the cos factor, m = 1. Thus,
A r, =

0 I 4
4 a

0 dk r 2 dr d cos sin cos r a coskzI 1 k < K 1 k >

A r, = 0 aI dk coskzI 1 k < K 1 k >


0

where < > is the smaller (larger) of a and .


b) From problem 3.16 b),

m l r l P 1l cos

1
=
x |
|x

with
ml =

0 I
4 a

J r ,
d 3 x Pl cos r l+1
1

A similar expression can be written on the outside by redefining r < and r > .

0 dke im J m kJ m k e k|z|

m=

Note z = 0, and = 0, so
A =

0 Ia
2

0 dke k|z|J 1 kJ 1 ka

5.16

On the inside, at the center of the loop


loop
=
H
A 1 r cos
loop at the center of the loop, which is directed in the z direction.
From Eq. (5.40), we are given

a) The system is shown in the figure

H z = 10 B A 1
If >> 0
A 1 1 m3
4 b
and from (5.40), at r = 0
2
3
H z = I + 1 m3 = I + I a 3 = I 1 + a 3
2a
4 b
2a
4 b
2a
2b

I shall use the magnetic potential approach and will call inside the sphere region 1 and outside the
sphere region 2.
1 = loop + A l r l P l
l

2 = loop + B l r l1 P l
l

, and we have the boundary conditions,


=
where H
H 1|| = H 2|| 1 r = b = 2 r = b
0 1 r = b = 2 r = b
r
r
We are given that b >> a, so

loop = 1 m cos
4 r 2
with m = a 2 I. (From the form of loop, only the l = 1 term contributes.) The boundary
conditions give
A 1 b 1 = B 1 b 11

2 0 m
2m
+ 0A1 =
2B 1 b 3
4b 3
4b 3

So
0
A 1 = 2 m3
4 b 2 + 0

5.18

F z = z mB z
with m = a 2 I, and (from Eq. (5.64))
Bz =

a) From the results of Problem 5.17, we can replace the problem stated by the system

0
4

2m
z3

with m = a 2 I , and z = 2d
Fz =

0
2a 2 I a 2 I 3 4
4
2d

with agrees with out previous result.

where I is equidistant from the interface and is equal to I =


loop is a. Now from Eq. (5.7)

r 1
r +1

I. The radius of each current

on I = I dl Br
F
= dl B
r + dl B
= dlB r + dlB r
dl B
By symmetry, only the z component survives, so, from the figure
z = dlB r
dl B

a
4d 2 + a 2

2d
4d 2 + a 2

+ dlB

So
Fz =

2aI aB + 2dB
r

4d 2 + a 2

with B r and B given by Eqs. (5.48) and (5.49) and cos =

2d
4d 2 +a 2

,r=

4d 2 + a 2 , and I I .

c) To determine the limiting term, simply let r 2d and take the lowest non-vanishing term in the
expansion of the magnetic flux density.
F z = aI aB r + 2dB
d
0I a a
F z = aI a
d
4d 2d 2
Fz

+ 2d

0I a2
4

1
2d 3

a
2d

3 0 a 4 I I
32
d4

The minus sign shows the force is attractive if I and I are in the same direction. This same result
can be gotten more directly, using

3 0 a 4 I I
32
d4

5,19
Hz =
The system is described by

M0
2

zL
a 2 + L z 2

z
+
a 2 + z 2

with a similar expression below the cylinder.


= 0 H
+M

B
Thus inside the cylinder,
z

a 2 + z 2

Lz
a 2 + L z 2

0M0
2

z
+
a 2 + z 2

Lz
a 2 + L z 2

0M0
2

a 2 + z 2

zL
a 2 + L z 2

Bz = 0 M0
2

Bz =

+ M0

while above the cylinder,

is constant within the cylinder. The


The effective volume magnetic charge density is zero, since M
from Eq. (5.99)) is M 0 , on the top surface and M 0 on the
effective surface charge density (n M
bottom surface. From the bottom surface the potential is (for z > 0
a
d
M
= 0
b = 1 M 0 2
4
2
0 2 + z 2 1/2

Bz =

First we plot B z in units of a for L = 5a

a 2 + z 2 z

By symmetry, the potential from the top surface is (on the inside)
t = M0
2

a 2 + L z 2

1
2

1+z 2

1
2

1+z 2

5z

gz =

if z < 5

1+5z 2

if 5 < z

z5
1+5z 2

L z
gz

The total magnetic potential is


= b + t =

M0
2

M0
2

a 2 + z 2 z +

a 2 + L z 2

L z

0.8

So, on the inside of the cylinder,


M
Hz = 0
2
z

0.6

M0
2

a 2 + L z 2

a 2 + z 2

Lz
a 2 + L z 2

a 2 + z 2 z +

L z

0.4
0.2

Hz = M0
2

10

And similarly, H z in units of a for L = 5a.

while above the cylinder,

5.26
12 2
fz =
12

z
1+z 2

5z

z
1+z 2

1+5z 2

if z < 5
The system is described by

if 5 < z

z5
1+5z 2

fz

0.4
0.2

10

-0.2
-0.4

Since the wires are nonpermeable, = 0 . The system is made of parts with cylindrical
symmetry, so we can determine B using Amperes law.

B dl = 0 J da

= 0
J, or
B
On the outside of each wire,

B dl = B2 = 0 I B out =

0I
2

On the inside of each wire

B dl = B2 = 0 I R 2 ,
2

B in =

0I
with R = a, b
2 R 2

From the right-hand rule, the B from each wire is in the direction. From the above figure, using
is in the z direction. Since
=B
,
the general expression for the vector potential, we see A
A
B z = A z A z = B z d

Thus
Az =

20 I ln

+C

0I 2
4 R 2

= 40 I ln

2
R2

on the inside

+ 1 on the outside

where Ive determined C = 1/2, from the requirement that A z be continuous at = R. Let l be the

length of the wire. Then we know the total potential energy is given by
W= 1
2
Consider the second term

J Ad 3 x =

5.27

l J Ada + J Ada
a
a
b
b
2

J b Ada b . The system is pictured as

l
2

The system is described by

From the figure


a =
b,

d+
so, since J b =

2a = d 2 + 2b 2d b cos

I
b 2

l
2

J b Ada b =

A out a + A in b b d b d

l I
2 b 2

I
= l I2 0
2 b 4

ln

2a
2

2b
2

+1

b d b d

2
I
= l I 2 0 2 1 b 2 1 + 2 ln d 2
2 b 4
4
a
l
2

we get

= l
2

b d b
B=

0
4

1 + 2 ln d I 2
a
2

J a Ada a is equal to
l
2

B dl = 0 I enclosed
0I
,<b
2 b 2

B=

b
2
2
I
l I 2 0 2 ln d 2 + 1 2b
2 b 4
0
a
b

The first term

Using Amperes law in integral form

0I 1
,b<<a
2
B = 0, > a

Now the energy in the magnetic field is given by ( l is the length of the wires)

J a Ada a =

0
4

l
2

1 + 2 ln d I 2
b
2

W= 1
2

B H d 3 x =

1
2 0

B2d3x

Thus
0
4

W= l
2

2
1 + 2 ln d I 2 = l L I 2
ab
2 l

or

0I
2

1
2 0
=

L = 0 1 + 2 ln d 2
l
4
ab

1
2 0

l 2
0I
2

b
0

b2

d + 2

l 1 + 2 ln a
b
2

L = 0
l
4

a
b

= l L I2
2 l

1 + 2 ln a
b
2

If the inner wire is hollow, B = 0, < b, so


L = 0 ln a
l
2 b

5.29

C = l = 2
2
12
l
ln dab
The system is described by

Thus
L C =
l
l

This problem is very much like 5.26, except the wires are superconducting. We know from section
5.13 that the magnetic field within a superconductor is zero. We will be using
W= 1
2

J Ad 3 x =

l J a Ada a + J b Ada b
2

Using the same arguments as applied in problem 5.26,


I
2
ln

Az =

I
= 4
ln

+C
0,

Thus if we consider the second term


l
2

l
2

2
R2

J b Ada b ,

J b Ada b =

A out a + A in b b d b d

l I
2 b 2

b
2
I
l I2
2 ln d 2 b d b = l
2 b 4
2
0
a

The first term

l
2

+ 0 on the outside

on the inside

2 ln da I 2

J a Ada a is equal to
l
2

J a Ada a =

l
2

2 ln d I 2
b

Thus
W= l
2

2
2 ln d I 2 = l L I 2
ab
2 l

so
L =
l

2
2 ln d
ab

Now using the methods of problem 1.6, assuming the left wire has charge Q, and the right wire charge
Q, we find
12 =

da

Edr =

Q
l

da

2
1 + 1
dr l ln d
r
2
ab
dr

2
2 ln d
ab

2
=
2
ln dab

ytRZ{BU F$st8$t +$st

_YkR${R x(xU 7BjZ$BtR B VRR$t8t '

_BAj8 xE

=R$t Y }$t{$}j BP RZ}}BR$$Bt Y 8st${ hjI s

bY

c ) 8$twy, D

stI

n wnD )
\

$t Y {s%$k $R JZsj B Ys BP s {BtIZ{B b$YBZ Y

( y

n tBYBj wnD )
4
| $R Y Zt$ %{B sjBt Y
bY *

*I${$Bt

({ |
*)

1$'

stI

bY

'

stI

1$'

*| )

{st A {sj{ZjsI PB8 V8}:R jsbU

( O'

5
Vd,d w, *D ) 

VfE w, *D

*|I

VfE w, *D

Qn A Y %{B

|
( O w' *

n
M$

R$t 

7*

n wnD )
\

( y

ytR$I Y R}Y

FI${$BtD

stI tB$t F
|

c )

stI

n wnD )
\

'| ) *| b U

{

$ wy 1  K 1 D

M$
n

( yw

r?| L ?r|D ) nd

({

 K1 D F| Q

1 $ w y1

n wnD )
\
n wnD ) n \
n)
4

M$
n

p\Vd,d wI , *I Di

c |
n
( y R$t  1 *

 R$t *?| L {BR *r|D ) nd

( y

|)
( yF

F I${$Bt

( y

1
n

|
( y R$t  * )
1

( y

n n

( y

n
c

!ZR$I Y R}Y

stI

k ) U

d
n
; R$t  |
* ) ( y; n
1
n

n|
w|

R$t 
; 1 {BR 
| L n | ) (
n

;$

N En D  En
n

_BAj8 xdn

Bs$Bt s-$R Y RZPs{ {Zt ItR$k

Fs-$R

E
n ) $yn wy 
nD

| $R Y Zt$ %{B sjBt Y


bY *

_BAj8 xdM

wsD 3B8 Y RZj BP _BA xd Y 8st${ hjI s Y {Zt jBB} {st A {sj{ZjsI Ak }js{$t Y 8I$Z8
b$Y st $8s {Zt BP 8st$ZI

*I${$Bt qYPB Y %{B }Bt$sj


- n nI
8
|I
d
w D I
R$t  I *
(
n
Qy )
Q I
( y
;$
;$
/n  nI/
/n  nI /

{I

n wnD )
\

|I sjBt h-I
qB {sk BZ Y sAB% $tsj b }B`{ *
Ys8Bt${R

?

stI

sjBt Y

n wnD ) n8 wnD ) y R$t *|

rI${$BtR

/n  n / $t R}Y${sj

stI -}stI db

yt s R}Y${sj {BBI$ts RkR8 b$Y $R ?


r }jst IhtI Ak Y $8s$t {Zt jBB} $R B$$t s Y {t BP
Y jBB} stI $R F s-$R }B$t$t B Y {Zt jBB} {  Y 8st${ hjI IZ B Y $8s$t {Zt $R $%t Ak eJ

wx;MD stI wx;D V Y jB{s$Bt BP Y {Zt jBB}

*|I )  R$t *I ?| L {BR *I r|

y,

0
y 1 L ;Q 1 ,

{BR  )

1Q
y1 L ;Q1

qYPB

stI Y PB{

4

qY PB{ Bt Y {Zt jBB}U

Bn
WB Ys ABY

stI

4

d
1

;
d

{ Iy

Q ( y

$

(y

| )  R$t  F
Qf
|

{{ I

{ *| w4 | L 4 |DQf )

|
w{4 

Qf ) 1$y R$t F|,

|Qf ) {BR F|

{4

|
Qf

 {4

3 wdDz w1z L dD


1z

z )(

Y PB{ $R IB8$tsI Ak Y

Bn

$
)

$

(y

$

(y

y1zLd

w y1 L ; Q 1 D 1

wy 1 L ;Q 1 D1

w y1 L ;Q1 D1

qY PB{ $R ss{$% Vjts$%jk PB


8B8tR

LQ

F ) y1 L ;Q1


L

F|

y;
|
;F
( Q

Fw|
F
( n|

l5

-(

-(

- wF D )

l5

-(

n wF D )
4

Fn

l5

 d1

En N 4n )  ;$( nw|F N En Dw|F NFEn D  En N E


nI

nI

 1$( EE
Fn

y;
F|
Q;

n wF D )
4

n
(I )

 d1

U
1

0
'I1 L wF  Ub1D1

D1

 /F  U1 / 

( -(

0

'I1 L wF L Ub1D1

M
5
U
U
y 1 L w F  D 1 L /F L /
1

M
5
U
U
y 1 L w F  D 1  y 1 L w F  D1 L U
U
1

M
5
U
y 1 L w F  D1  U
D1 

F L Ub1
y1 L wF L Ub1D1

0

F  Ub1
0
y1 L wF  Ub1D1
l

( -(

F  Ub1
y1 L wF  Ub1D1

F  Ub1
y1 L wF  Ub1D1

0

0
;

cFc

Fc

F|

L1

U
1

U
1

$tR$I Y {kj$tI

M
F|

F L Ub1
y1 L wF L Ub1D1

F L Ub1
y1 L wF L Ub1D1

F L Ub1
y1 L wF L Ub1D1

0

1

PB

Fk
U

PB

PB

M
5
U
y 1 L w F  D 1 L 1F
D1 

F  Ub1
0
y1 L wF  Ub1D1

$R Y R}ss$Bt Abt Y bB I$}BjR ]BtRJZtjk Y }Bt$sj tkU

$ ({ 1

n N -n Q+ I L d - -n N znI QyI
;$
/n  nI/
/n  nI /

l
Q'I

y 1 L wF 

F s-$RU

n LnD)
( wI

y1 L wF 

Fs-$RU
l
-(

n wFD )  ( - wF D F| ) 
I
(F

qY 8st${ hjIR sjBt Y

N En I D  En I

n
n1

l5

y 1 L wF 

n wFD )  ( - wF D F| )  -(
I
(F
1

n I ) { Iw$y1 D|
E
F

;$

-(
w1$ D
;$

qY sZ-$j$sk hjIR sjBt Y

n IU
IZ B E
n)
4

-(

- w F D )

- w F D )

y1 L ;Q1

F|  

8
8
n N znI I
d
d
-(
-(
Qy )
QyI 
QyI
;$ B} /n
;$ ABB8 /n
/n  nI /
 nI/
 nI /

;$

- wF D )

qYPB

Q ( y ABY {Zt jBB}R {st A s}}B-$8sI sR }B$t I$}BjR b$Y I$}Bj


E
n ) { w$y1 D|F,

E
n

F s-$R

VjBt Y

\y {BR  L Q R$t DiF|

1Q
y$ 1
y L ;Q 1

d
;$

n N

Qhdd w{BR D F|

y;

w y1 L ; Q 1 D 1

n ) ( yt Y {kj$tI${sj {BBI$ts RkR8 b$Y $R B$$t s Y


qY &{$% 8st${ %BjZ8 {Ys '- )
{t BP Y {kj$tI stI $R F s-$R sjBt Y s-$R BP Y {kj$tI $t Y 8st$9s$Bt I${$Bt

Qf ) 1$y {BR F|

Q
y
h
h1dzLd w{BR D F|
w{BR  D 
z 1zLd
w z L dD

y; Q
n
I
) n$ ( {{
F|   $ ( { 1
n1
wy1 L ;Q1 Dxb1

qY 8st${ hjI s

D )
- wn

\yhd w{BR D 

_BAj8 xd

z ) ( 8U

{{ I

( {{

qY $R tB P {Zt qYPB Y R{sjs }Bt$sj s}}Bs{Y $R s}}j${sAj

F
|Qf ) 1$y{ w4 R$t  L 4 {BR D|

wy1 L ;Q1 Dz L1

1 I
( y {{

{{ I

 1n $

sR b$Y Y RZj PB8 wsD

- wn
D )

nJ )  (J
F|
/ $ 3 3 F| )  n1$( wy1 EE
(F F) y L;Q
L ;Q1 D1

 {4 |DQf

Bn

3 wdDz w1z L dD


y1zLd
h1dzLd w{BR D
1z wz L dD
wy1 L ;Q1 Dz Ln b1
z)(

{ Iy

s {BtRstR BP Y $ts$Bt stI Ys

Bn

3 wdDz w1z L dD


y1zLd
h1zLd w{BR D
1z z
wy1 L ;Q1 DzLnb1
z )(

n )
{n 4Qf

qY PB{ s{$t Bt Y jBB}U

bY

qY$R $R Y hjI IZ B s 8st${ I$}Bj

qY Bs$t RZPs{ {YsR RZj RZPs{ {ZtR yt Y R}Y${sj {BBI$ts RkR8 b$Y Y

w{D 3B

k
F
8
wI , *I DV wI , *IDQ I ) R$t  R$t *& &
{E VfE w, *D VfE
d ,d
f,d E,d

n wnD ) n \
n)
4

Qn

n {s%$k wnD )
4

qY hjI $tR$I Y R}Y $R Zt$PB8 stI }B$t B

Bt Y {kj$tIU

R$t  {BR * )

yU

qY hjI $R Zt$PB8 stI $t Y I${$Bt }}tI${Zjs B Y j$t `B$tt$t Y s-R BP Y {kj$tI stI Y YBj yt
8R BP Y {Zt

]BtRJZtjk Y hjI $tR$I Y {s%$kU


|
(O F

c
d
& & R$t w
fLd f,d E,d
1f L d k

fE

'*|  ' *|I ) F| Qn

Vd,d U

{E\Vd,d wI , *IDi

M$

5
F
k
8
M$

I I
I
I
I

I I
I I
I
VfE
w , * D R$t  {BR * Q ) 
w , * D Vd,d w , * DQ
p VfE w, *D VfE
) R$t  {BR *&f,d &E,d

'n  'nI ) Qn

n {s%$k wnD )
4

R$t  R$t * )

wI , *ID R$t I R$t *I Q I


VfE

PB8 Y {kj$tI s-$R B Y YBj s-$R b Ys%U

]BRR 8Zj$}jk$t Y sAB% JZs$Bt b$Y F


| wY Zt$ %{B sjBt Y

wI , *IDw R$t I R$t *I?


I
I |DQ I
VfE
| L R$t  {BR * r

VR RZjR BP Y sAB% Y %{B }Bt$sj $R

 ' *| D
I

stI Y $tsjR {st A {s$I BZ ZR$t Y BYBBtsj$k }B}$R BP R}Y${sj Ys8Bt${RU

O'*|

s 8sRZI b$Y R}{ B Y s-$R BP Y YBj qYPB

n {s%$k wnD )
4
N

$'1 O

WB Ys R$t  I R$t *I stI R$t  I {BR *I {st A b$t $t 8R BP

n tBYBj
4

$R Y %BjZ8 {Zt ItR$k 7$8$jsjk b Ys%

n YBj wnD )
4
I

s-$R

c
d
V w, *D
fLd fE
1f L d k

fE

n tBYBj wnD  4
n YBj wnD
n {s%$k wnD ) 4
4
Fs-$R sjBt Y {kj$tI

c I I
;$
V w , * DVfE w, *D
fLd fE
1f L d k

fE

) 8s-wy, D qY %{B }Bt$sj $R YPBU

YBj 8$tZR Ys BP s R8sjj {BtIZ{B hjj$t Y YBj b$Y Y Rs8 %BjZ8 {Zt ItR$$R $U

yt Y }Bjs {BBI$ts RkR8 b$Y Y

/n  n /

*Z IsU WB%8A d 1((d

More Problems for Chapter 5

BZR$I Y {kj$tI

M
F|

$tR$I Y {kj$tI

M
F|

BZR$I Y {kj$tI

ytRZ{BU F$st8$t +$st

More Problems for Chapter 5

_YkR${R x(xU 7BjZ$BtR B VRR$t8t 'M

_BAj8 xd(

/n  nI /

=RPZj $It$$RU

(Od w>'D
('

>

({d w>'D
('

\O( w>'D  O1 w>'Di

qY %{B }Bt$sj

*I

| $R Y Zt$ %{B sjBt Y


bY *

QfnI
/n  nI/

({

n wnD )
\

1wy1 L F 1 Dn b1

n
\
;$

Q>>>/F/ Od w>yD )

( {y

;$

1$
(

*|I
Q*I
/n  nI/

E)

;$

Q>7Ew** D OE w>'DOE w>yD>/F/


I

Q>OE w>'DOE w>yD>/F/

3 8

( {y

wy1 L F 1 Dn b1

E)

/n  nI /

;$

$ E)

) 8$twy, 'D stI

1$
(

1$
(

'k

( {y

;$

( {y

( {y

\* w', FD )

Q>>/F/ Od w>yDOd w>'D

I
7Ew* * D

$ E)

Q> {BRw>FD{E w>'c DE w>'k D

) 8s-wy, 'D qY $tsjR

F
8
7Ew** D R$t *I Q*I ) {E 7E*

1$

F
8
7Ew** D {BR *IQ*I ) p 7E*

(
1$

e-}stR$Bt BP wsDU

4' w', F D ) 

Q> {BRw>FD{E w>'c DE w>'k D

n )n \
n )  (\* '| L d ( w'\* DF| ) 4' '| L 4F F|
4
(F
' ('

8
(

7wdED* Q*I

7wdED* Q*I

1$

7Ew** D w R$t *I?| L {BR *I r|DQ*I

4F w', FD )

!t Y

Fs-$R w' ) (D 'c

(\*
('

{E 7E* 1$&E,d

) 1$ R$t *&E,d

p 7E* 1$&E,d

n wnD )
\

3 8

( {y 1

$ E)

;$

' (

( {y

( {y y$

Q> {BRw>FD{d w>'c Dd w>'k DQ>*|

$

( {y

\* w', FD )

4F w', FD )

Q> {BRw>FD{d w>'c Dd w>'k D

(\*
('

4' w', FD ) 
(\*
('

'

\*

( {y

( {y

4' w(, FD )

Q>>/F/ Od w>yD j$8

( {y

( >/F/
)(

(F

' (

{y1
1

8I$Z8 Y sZ-$j$sk hjI

n
wg

n
N nD;Q+

wn N

Bn

n
I

b$Y 8st$9s$Bt

n
-

nD
wn -

4n

bY

n
w-

n
wg

Bn

8 +
)

n
4

8 +
~

8
~

bY

n
L-

Bt Y hR bB $tsjRU

Bn

n nzD ) w4n N nzDn


4n w-

n D4n Q+ L
wn N -

- +
L

n
L w4

n
w4

n
w-

N nzD4n Qy 

8
)

8
)

wn N

8
~

n D;Q+
n
g

n
wg

n D-Q+
n L
wn N 4

n D4n Q+ L
wn N n

n
w-

qY PB{ }B$tR B

n
I

BZR$I Y 8I$Z8 8ZR A

n
'- 4Q+

'-  -

n
- 4Qy

s Y &{$% %BjZ8 stI RZPs{ 8st${ {Ys

) (,

-

n N nz
-

n I${$Bt
Fs-$R sjBt Y Fs-$R $R $%t Ak w_BA xd RY$P$t Fs-$R Ak Ub1DU
l
M
F
FLU
$
0
F|
(y1 L F 1
y1 L wF L UD1

n Dnz
N-

( \-

F| I${$Bt

4 w' D )
bY

F
U $

VRRZ8$t Y As $R jBt stI tsBb Y PB{ $R Yt

$

y1 L U1

y1 L ;U1

F|  

d
1

1
|
( \- F

$ Y As $R ss{I B Y 8I$Z8

{
'
$K1

w(

n
w4

n
N nzD-Qy

c ' c KD

 $R Y }8sA$j$k BP Y {BtIZ{B

n
N nzD;Qy

N nzD4n Qy

\DU

n  \- \4 wUD  4w(DiF|
N nzD4Qy

3B8 Y V8}:R jsb b {st {sj{Zjs Y 8st${ hjIR $t Y Y $BtRU

n 
N nzD-Qy

1 )

n
w-

_BAj8 x1

n Dnz  w4n N nzDn Qy


N-

n
w4

n N 4n D
wn 4nD  nw-

) ( $t Y $Bt BP $tR 7B

n
N nD;Q+

Od w>'D

$R {Bt$tZBZR s Y RZPs{

n
) n N -

$R Y &{$% sI$ZR BP Y As w$y

- +


n N nzDn Q+ 
n Qy
n N nD4n L w4n N nDw4
-

n
wg

$R Y hjI IZ B Y $8s As 8st

n wF D )  d
4

n nzD 4n Qy
w-


n N nD4n L w4n N nDn  n wn N 4n D Q+
-

)
=R$t Y $It$k

'

Y 8st${ hjI IZ B Y $8s As sjBt Y

n
N nzD;Qy

n D 4n ) wn N nD4n L w4n N nDn  nwn N 4nD


-

nzD 4n

yt s {BBI$ts RkR8 b$Y $R B$$t s Y `B$t Abt Y As stI Y $8s stI $R

VjRB

qY PB{

ItR$$R R}{$%jkU

n D ) wn N nD4n L w4n N nDn


4n wn -

wn

Bn

Bn

n 4n

(
('

3B8 Y RZj BP _BA x1( Y PB{ Bt Y As 8st $R $%t Ak

=R$t Y }BIZ{ ZjU

$R st -tsj hjI b Ys%

n
nz I

%st$RYR 7$t{

n D 4n Q+
wn -

( >/F/

(F

{BtIZ{B s 9B }Bt$sj G {st ZR Y 8YBI BP $8s stI }js{ Y 8I$Z8 b$Y st $8s As 8st

F1 Dnb1

n D;Q+
n
g

wsD qY PB{ Bt Y ABZtI %BjZ8 stI RZPs{ {ZtR

d
wy1 L F 1 Dn b1

qYPB Y$R }BAj8 {st A sI j$, Y stsjBBZR j{BRs${ }BAj8R b$Y s }Bjs$9I As ss$tR s }P{

d
w y1 L

8
)

}}tI${Zjs B Y RZPs{ ]BtRJZtjk Y RZPs{ $R s JZ$}Bt$sj stI b {st R $R }Bt$sj B A 9B

'-

\{d w>'Dd w>yDi

7$t{ Y $R tB P {Zt Y 8st${ R{sjs }Bt$sj s}}Bs{Y $R s}}j${sAj G$Y$t Y $tht$ }8sAj

k
d
(
L
Od w>'D
(' '

_BAj8 x1(
=RPZj $It$kU

'

_BAj8 x11

Q>>>/F/ Od w>yD

\{d w>'c Dd w>'k Di

Q>Od w>yDOd w(D

( {y

4F w(, F D )

>/F/ Od w>yD

Fs-$R ' ) (U

'

Q>Od w>yDOd w>'D

( {y

(
('

( {y

F1 Dnb1

1 w y1 L

Q>> {BRw>FDd w>yD

e-}stR$Bt BP wADU

( {y

(
('

Q>> R$tw>FD{d w(Dd w>yD ) (

Q> {BRw>FD j$8

) 1$ {BR *&E,d

Q> {BRw>FD{E w>'c DE w>'k Dw1$&E,d Dw R$t *?| L {BR *r|D

( {y

4F w(, FD )

F
Q> {BRw>FD

( {y

Q>> R$tw>FD{d w>'c Dd w>'k D

( {y

4' w(, FD )

yU

( {y

\* )

'

'k

) ( stI

(\*
(F

| Y %{B }Bt$sj $R
WB$t w R$t *?
| L {BR *r
|D ) *

4n

Q>Od w>'DOd w>yD>/F/ *|

$

I| I
 R$t *I ?
| L {BR * r
Q*
/n  nI /

1$
(

FI ) (U

3 8

( {y 1

n wnD )
\

7$t{

7Ew** D w R$t *I?| L {BR *I r|DQ*I

1$
(

w{D

wsD =R$t Y -}stR$Bt BP eJ wnd;MD b$Y

!t Y

Q>OE w>'DOE w>yD>/F/ w1$&E,d Dw R$t ?| L {BR r|D

E)

*|I )  R$t *I ?| L {BR *I r|

n wnD )
\

'c

;$

3 8

3 8

( {y

n wnD )
\

>
 \( w>'D L 1 w>'Di

I${$BtU

]BtRJZtjk

bY

(d w>'D
('

\{( w>'D L {1 w>'Di

y$

Q>> {BRw>FDd w>yD )

>

'I ) y, F I ) (U

wAD =R$t Y -}stR$Bt BP _BA ndEwAD stI tB$t

*Z IsU WB%8A M 1((d

d
1 

d
1

U{ 1

N 1$

8 KF
(

U }

1$'

wK

c ' c yD

4 w' D ) (

w'

n $R sjBt Y *| qY 8st${ tk


qY 4
8K
8 y
d
d
41 'Q'Q* L
41 'Q'Q*

1 

{

1$K1

)
qY RjPT$tIZ{st{

({

4 w' D )

1 (

'

k1

'Q' L

{1 +

;$

d
1 (

( jt

N 1$

y
K

Zt$ jtY

U)

d
;$


;

( jt

y
K

8 yF
K

({

1$'

k1

'Q'

k yD
} Zt$ jtY

3B Y {sR BP

 $ R$8}j$hR BU
U)

d
y
(
L jt
;$ ;
K

yP Y $tt {BtIZ{B $R s Y$t YBjjBb ZA Y b$jj A tB 8st${ hjI $tR$I Y YBjjBb ZA ]BtRJZtjk Y
RjPT$tIZ{st{ } Zt$ jtY bBZjI AU
(

U)

;$

Solutions to Problems in Jackson,


Classical Electrodynamics, Third Edition

y
K

jt

Homer Reid

_BAj8 x1

ytR$I st $Isj {BtIZ{B Y j{${ stI 8st${ hjIR %st$RY 3ZY8B sjj {YsR stI {ZtR s Bt Y
{BtIZ{B RZPs{R yt s {kj$tI${sj {BBI$ts RkR8 b$Y Y Fs-$R sjBt Y I${$Bt BP Y bB {BtIZ{BR Y

October 8, 2000

RZPs{ {Ys stI {Zt ItR$$R s F$tI}tIt N d w', *D stI 1 w', *D A Y RZPs{ {Ys ItR$$R Bt
Y bB {BtIZ{BR Y RZPs{ {Zt ItR$$R s Yt d w', *D8 stI 1 w', *D8 sjBt Y FI${$Bt bY 8 $R
Y R}I BP Y {YsR ]BtRJZtjk Y R{sjs stI %{B }Bt$sjR sU
d
w', *D )
;$6

\F w', *D )

l8
;$

l8

d w'I , *I D I
Qyd L
/n
 nI /

d w'I, *I D8 I
Qyd L
/n
 nI /

1 w'I , *I D I
Qy1
/n
 nI /

1 w'I , *I D8 I
Qy1
/n
 nI/

Chapter 4: Problems 8-13

Problem 4.8

M
) 68 w', *D

bY QyId stI QyI1 $ts B% Y RZPs{R BP Y bB {BtIZ{BR WB Ys \' ) \* ) ( N R A Y Bsj
{Ys } Zt$ jtY Bt Y bB {BtIZ{BR Yt b Ys% R8 ) {  qYPB

A very long, right circular, cylindrical shell of dielectric constant /0 and inner and
outer radii a and b, respectively, is placed in a previously uniform electric eld E 0
with its axis perpendicular to the eld. The medium inside and outside the cylinder
has a dielectric constant of unity.

{
\F w', *D ) 6 w', *D
R

(a) Determine the potential and electric elds in the three regions, neglecting end
eects.

qY j{${ stI 8st${ hjIR s

n /1 )
4 1 ) /n \

(b) Sketch the lines of force for a typical case of b 2a.

F ( k1 F d ( k1

o 1 ) \n i1 )

('

F d (\ k1 F (\ k1 F
F

' (*

(c) Discuss the limiting forms of your solution appropriate for a solid dielectric
cylinder in a uniform eld, and a cylindrical cavity in a uniform dielectric.

' (*

('

{
R

k1

We will take the axis of the cylinder to be the z axis and the electric eld to
be aligned with the x axis: E0 = E0i. Since the cylinder is very long and were
told to neglect end eects, we can ignore the z direction altogether and treat
this as a two-dimensional problem.

o1

qY t$R $t j{${ stI 8st${ hjIR } Zt$ jtY sU


Po )

P4 )

6
d R1
)
1g
1

d 1
d
U{ )
1
1

(a) The general solution of the Laplace equation in two dimensional polar coordinates is

(r, ) =
[An rn + Bn rn ][Cn sin(n) + Dn cos(n)]

o 1 Q+

4 1 Q+ )

d 1 {1
6 1
1
R

o 1 Q+

For the region inside the shell (r < a), the B coecients must vanish to keep
the potential from blowing up at the origin. Also, in the region outside the shell

qs,$t Y s$B BP P4 stI Po jsIR B Y htsj RZjU


Ug ) 6

Homer Reids Solutions to Jackson Problems: Chapter 4

(r > b), the only positive power of r in the sum must be that which gives rise
to the external electric eld, i.e. E0 r cos with An = 0 for n > 1. With these
observations we may write expressions for the potential in the three regions:

(r, ) =




rn [An sin n + Bn cos n],


n

r [Cn sin n + Dn cos n] + r [En sin n + Fn cos n],



E0 r cos +
rn [Gn sin n + Hn cos ],

Homer Reids Solutions to Jackson Problems: Chapter 4


which leads to

r<a

r>b

The normal boundary condition at r = a is






0
=
r x=a
r x=a+

Gn = Cn b2n + En

0  n1
[An sin n + Bn cos n] =
na


nan1 [Cn sin n + Dn cos n] na(n+1) [En sin n + Fn cos n]
From this we obtain two equations:

nan [An cos n Bn sin n] =

0
B1 = D1 F1 a2


D1 =

0
1
1+
B1
2


F1 =

0
1 2
a 1
B1 .
2


(2)
B1 = D1 + F1 a2
and



D1 b2 F1
0
0
2
H1 = b E0 + D1 b + F1





0 = 2b2 E0 + b2 1 +
D1 + 1
F1
0
0

H1 = b2 E0 +
2

nan [Cn cos n Dn sin n] + nan [En cos n Fn sin n]

from which we obtain two more equations:

Substituting from above,

An = Cn + En a2n

(3)

Bn = Dn + Fn a2n

(4)

Similarly, from the normal boundary condition at r = b we obtain


0
0  (n+1)
E0 cos
[Gn sin n + Hn cos ] =
nb



nbn1 [Cn sin n + Dn cos n] nb(n+1) [En sin n + Fn cos ]

(7)
(8)

The four equations (1), (3), (5), and (7) specify a degenerate system of linear
equations, which can only be satised by taking An = Cn = En = Gn = 0 for
all n. Next, for n = 1, the system of equations (2), (4), (6), and (8) specify the
same degenerate system of equations, so Bn = Dn = Fn = Gn = 0 for n = 0.
However, for n = 1, we have

(1)

Next, the tangential boundary condition at r = a is






=
x=a+
x=a
or


(6)

giving

b2 E0 n1 + Hn = Dn b2n + Fn .

0
An = Cn En a2n

0
Bn = Dn Fn a2n


(5)

Finally, we have the tangential boundary condition at r = b:



bE0 sin +
nbn [Gn cos n Hn sin n] =

nbn [Cn cos n Dn sin n] + nbn [En cos n Fn sin n]

or


Gn = Cn b2n En

0
0 2
b E0 n1 Hn = Dn b2n Fn


0

a<r<b

4b2 E0 =
or
B1 =


1 2
b ( + 0 )2 a2 ( 0 )2 B1
0
40b2
E0 .
b2 ( + 0 )2 a2 ( 0 )2

Homer Reids Solutions to Jackson Problems: Chapter 4

Homer Reids Solutions to Jackson Problems: Chapter 4

Then
20 ( + 0 )b2
E0
b2 ( + 0 )2 a2 ( 0 )2
20 ( 0 )a2 b2
E0
F1 = 2
b ( + 0 )2 a2 ( 0 )2
b2 (b2 a2 )(20 2 )
H1 = 2
E0 .
b ( +  )2 a2 (  )2

D1 =

The potential is

40 b2

b2 ( + 0 )2 a2 ( 0 )2 E0 rcos ,

20 b2
a2
(r, ) =
( + 0 )r + ( 0 )
E0 cos ,
b2 ( + 0 )2 a2 ( 0 )2
r

2
2
2
2
2

(b a )(0  )
b

E0 cos E0 rcos ,
b2 ( + 0 )2 a2 ( 0 )2 r

r<a
a<r<b
b < r.

As  0 , E0 r cos in all three regions, which is reassuring.


The electric eld is

40 b2

E0 [cos
r sin ]
,
r<a

b2 ( + 0 )2 a2 ( 0 )2




20 b
a2

r
( + 0 ) ( 0 ) 2 E0 cos

2
2
2
2

b ( + 0 ) a ( 0 )


a2
E(r, ) =
,
a<r<b
( + 0 ) + ( 0 ) 2 E0 sin

(b2 a2 )(20 2 )
b

E
[cos

r
+
sin

b2 ( + 0 )2 a2 ( 0 )2
r

+E0 [cos
r sin ]
,
b < r.
(b) In Figure 4.1 Ive plotted the eld lines for b = 2a,  = 50 . Also, as an
appendix to this document Ive included the C program I wrote to generate this
plot.

Figure 1: Field lines in Problem 4.8 for b = 2a,  = 50 .

b , in which case the eld becomes

40

( +  )2 E0 i,
0
E(r, ) =
20 ( 0 )  a 2
20

E0 i
E0 [cos
r + sin ],

( + 0 )
( + 0 )2
r

r<a
r > a.

(c) For a solid dielectric cylinder in a uniform eld, we would have a 0. In


that case the eld would look like
2
0

r<b

 + 0 E0 i,

2
E(r, ) =
2

2 ) b
(

0
E0i
E0 [cos
r + sin ],

r>b
2
( + 0 )
r
On the other hand, a cylindrical cavity in a uniform dielectric corresponds to

Homer Reids Solutions to Jackson Problems: Chapter 4

Problem 4.9
A point charge q is located in free space a distance d away from the center of a
dielectric sphere of radius a (a < d) and dielectric constant /0 .
(a) Find the potential at all points in space as an expansion in spherical harmonics.
(b) Calculate the rectangular components of the electric eld near the center of
the sphere.
(c) Verify that, in the limit /0 , your result is the same as that for the
conducting sphere.
We will take the origin of coordinates at the center of the sphere, and put
the point charge on the z axis at z = +h. Then the problem has azimuthal
symmetry.
(a) Since there is no free charge within the sphere, D = 0 there. But since the
permittivity is uniform within the sphere, we may also write (D/) = E = 0
there. This means that polarization charge only exists on the surface of the
sphere, so within the sphere the potential satises the normal Laplace equation,
whence

(r, ) =
Al rl Pl (cos )
(r < a).

Homer Reids Solutions to Jackson Problems: Chapter 4


The normal boundary condition at r = a is




= 0

r r=a
r r=a+


lqal1
lAl al1 = (l + 1)Bl a(l+2) +
0
40 dl+1


0 (l + 1)
q

Al =
Bl a(2l+1) +

l
40 dl+1

On the other hand, 2 is just the potential due to a point charge at z = d:

q  rl

Pl (cos ),
r<d

40
dl+1
2 (r, ) =
(9)
 dl

q
P
(cos
),
r > d.
l
40
rl+1
Putting this all together we may write the potential in the three regions as


r<a
Al rl Pl (cos ),





rl
q
(l+1)
r
+
(cos
),
a
<
r<d
P
B
l
l
(r, ) =
40 dl+1


qdl

(l+1)

Pl (cos ),
r > d.
r
Bl +
40

(10)

The tangential boundary condition at r = a is






=
r=a
r=a+
al
q
40 d(l+1)
q a2l+1

40 dl+1

Al al = Bl a(l+1) +

Bl = Al a

2l+1

(11)

Combining (10) and (11), we obtain


Al =


0

1
+ l+1
l


0

1
+ l+1
l

Now, in the region r > a, the potential may be written as the sum of two
components 1 and 2 , where 1 comes from the polarization charge on the
surface of the sphere, while 2 comes from the external point charge. Since
1 satises the Laplace equation for r > a, we may expand it in Legendre
polynomials:

Bl r(l+1) Pl (cos )
(r > a).
1 (r, ) =

Bl =


2l + 1
q
l
40 dl+1


qa2l+1

1
0 40 dl+1

In particular, as /0 we have


Al 0
as must happen, since the eld within a conducting sphere vanishes; and
Bl

qa2l+1
.
40 dl+1

(12)

With the coecients (12), the potential outside the sphere due to the polarization charge at the sphere boundary is
1 (r, ) =

l
1
1  qa  a2

Pl (cos ).
40
d
d
rl+1

Comparing with (9) we see that this is just the potential of a charge qa/d on
the z axis at z = a2 /d. This is just the size and position of the image charge
we found in Chapter 2 for a point charge outside a conducting sphere.

Homer Reids Solutions to Jackson Problems: Chapter 4

(b) Near the origin, we have

Homer Reids Solutions to Jackson Problems: Chapter 4


must have

(r, ) = A1 rP1 (cos ) + A2 r2 P2 (cos ) +






30
q
1
50
=
z+
(z 2 x2 y 2 ) +
40 d2 ( + 20 )
2 d3 (2 + 30 )

so the eld components are


q
50  x
+

40 d2 2 + 30 d
q
50  y
Ey =
+

40 d2 2 + 30 d


q
50  z
30
+
Ez =
+
40 d2  + 20
2 + 30 d




 

Al Cl Pl (0)rl +
Bl Dl Pl (0)rl+1 = 0
0
0

l [Al Cl ] P (0)rl1 (l + 1) [Bl Dl ] Pl (0)rl+2 = 0.

Two concentric conducting spheres of inner and outer radii a and b, respectively,
carry charges Q. The empty space between the spheres is half-lled by a hemispherical shell of dielectric (of dielectric constant /0 ), as shown in the gure.
(a) Find the electric eld everywhere between the spheres.
(b) Calculate the surface-charge distribution on the inner sphere.
(c) Calculate the polarization-charge density induced on the surface of the dielectric
at r = a.
Well orient this problem such that the boundary between the dielectriclled space and the empty space is the xy plane. Then the region occupied
by the dielectric is the region a < r < b, 0 < < /2, and the problem has
azimuthal symmetry.
(a) Since the dielectric has uniform permittivity, all the polarization charge
exists on the boundary of the dielectric, so within its body we may take the
potential to be a solution of the normal Laplace equation. The potential in the
region between the spheres may then be written


[Al rl + Bl r(l+1) ]Pl (cos ),


0<<
2
(r, ) =


[Cl rl + Dl r(l+1) ]Pl (cos ),


<<
2
First lets apply the boundary conditions at the interface between the dielectric and free space. That region is described by = /2, a < r < b, and we

10

cylinders. For our Gaussian pillbox we take a disk of thickness dz and radius
r, a < r < b centered on the axis of the cylinders. By symmetry there is no
component of E normal to the top or bottom boundary surfaces, and the component normal to the side surfaces (the radial component) is uniform around
the disc. Hence

q
1
E dA = 2 r dzE =
= (2 a dz)
0
0
a
0 r

where is the surface charge on the inner conductor. This must integrate to
give the correct potential dierence between the conductors:
 b
a b
V =
E ()d =
ln
0
a
a
which tells us that, to establish a potential dierence V between the conductors,
the battery has to ow enough charge to establish a surface charge of magnitude
0 V
=
a ln(b/a)

(17)

on the cylinder faces (the surface charges are of opposite sign on the two cylinders).
It is useful to gure out the energy per unit length stored in the electric eld
between the cylinder plates here. This is just
 
1 b 2
Wv =
E D d d
2 a 0
 b
2
E () d
= 0
a

a2 2
ln(b/a)
0
0 V 2
=
ln(b/a)

(13)
(14)

Since these equations must be satised for all r in the region a < r < b, the
coecients of each power of r must vanish identically. In (13), this requirement
is automatically satised for l even, since Pl (0) vanishes for even l. Similarly,
(14) is automatically satised for l odd. For other cases the vanishing of the
coecients must be brought about by taking

Problem 4.10

E () =





= 0
=/2+
=/2




=
r =/2+
r =/2

which leads to

Ex =

Homer Reids Solutions to Jackson Problems: Chapter 4

(18)

where the v subscript stands for vacuum, since (18) is the energy per unit length
stored in the eld between the cylinders with just vacuum between them.
Now suppose we introduce a dielectric material between the cylinders. If the
voltage between the cylinders is kept at V , then the E eld must be just the
same as it was in the no-dielectric case, because this eld integrated from a to
b must still give the same potential dierence. However, in order to establish
this same E eld in the presence of the retarding eects of the dielectric, the
battery now has to establish a surface charge that is greater that it was before
by a factor (/0 ). With this greater charge on the electrodes, the D eld will
now be bigger by a factor (/0 ) than it was in our above calculation. So the


Al = Cl
0
Al = Cl


Bl = Dl ,
0
Bl = Dl ,

l odd

(15)

l even.

(16)

Next lets consider the charge at the surface of the inner sphere. There are
actually two components of this charge; one component comes from the surface
distribution of the free charge +Q that exists on the sphere, and the other
component comes from the bound polarization charge on the inner surface of
the dielectric

Problem 4.13
Two long, coaxial, cylindrical conducting surfaces of radii a and b are lowered
vertically into a liquid dielectric. If the liquid rises an average height h between the
electrodes when a potential dierence V is established between them, show that the
susceptibility of the liquid is
e =

(b2 a2 )gh ln(b/a)


0 V 2

where is the density of the liquid, g is the acceleration due to gravity, and the
susceptibility of air is neglected.
First lets work out what happens when a battery of xed voltage V is connected between two coaxial conducting cylinders with simple vacuum between
them. To begin, we can use Gauss law to determine the E eld between the

Homer Reids Solutions to Jackson Problems: Chapter 4

11

energy per unit length stored in the eld between the cylinders increases by a
factor (/0 1) over the result (18):
Wd = ( 0 )

V 2
.
ln(b/a)

On the other hand, to get to this point the battery has had to ow enough
charge to increase the surface charges to be of magnitude (/0 ) times greater
than (17). In doing this the internal energy of the battery decreases by an
amount equal to the work it had to do to ow the excess charge, namely
Wb = V dQ = V (2 a d) = ( 0 )

2V 2
ln(b/a)

(per unit length). The energy lost by the battery is twice that gained by the
dielectric, so the system with dielectric between the cylinders has lower overall
energy than the system with vacuum between the cylinders by a factor
W = ( 0 )

V 2
ln(b/a)

(19)

(per unit length).


Turning now to the situation in this problem, well take the axis of the
cylinders as the z axis, so that the surface of the liquid is parallel to the xy
plane. Well take the boundary between the liquid and the air above it to be at
z = 0. With no potential between the cylinder plates, the liquid between the
cylinders is at the same height as the liquid outside.
Now suppose a battery of xed potential V is connected between the two
cylinder plates. As we showed earlier, the combined system of battery and dielectric can lower its energy by having more of the dielectric rise up between the
cylinders. However, at some point the energy win we get from this is balanced
by the energy hit we take from the gravitational potential energy of having
the excess liquid rise higher between the cylinders. The height at which we no
longer gain by having more liquid between the cylinders is the height to which
the system will settle.
So suppose that, with a battery keeping a voltage V between the electrodes,
the liquid between the electrodes rises to a height h above the surface of the
liquid outside the electrodes. The decrease in electrostatic energy this aords
over the case with just vacuum lling that space is just (19) times the height,
i.e.
V 2
Ee = h( 0 )
(20)
ln(b/a)
This must be balanced by the gravitational potential energy Eg of the excess
liquid. Eg is easily calculated by noting that the area between the cylinders is
(b2 a2 ), so the mass of liquid contained in a height dh between the cylinders
is dm = (b2 a2 )dh, and if this mass is at a height h above the liquid surface
its excess gravitational energy is
dEg = (dm)gh = g(b2 a2 )hdh.

Homer Reids Solutions to Jackson Problems: Chapter 4

12

Integrating over the excess height of liquid between the cylinders,


Eg = g(b2 a2 )

h dh =

1
g(b2 a2 )h2 .
2

(21)

Solutions to Problems in Jackson,


Classical Electrodynamics, Third Edition

Comparing (20) to (21), we nd that the gravitational penalty of the excess


liquid just counterbalances the electrostatic energy reduction when
2( 0 )V 2
g(b2 a2 ) ln(b/a)
2e 0 V 2
=
g(b2 a2 ) ln(b/a)

Homer Reid

h=

November 8, 2000

Chapter 5: Problems 1-10

Solving for e ,
e =

gh(b2 a2 ) ln(b/a)
.
20 V 2

So I seem to be o by a factor of 2 somewhere.


Actually we should note one detail here. When the surface of the liquid
between the cylinders rises, the surface of the liquid outside the cylinders must
fall, since the total volume of the liquid is conserved. Hence there are really
two other contributions to the energy shift, namely, the change in gravitational
and electrostatic energies of the thin layer of liquid outside the cylinders that
falls away when the liquid rises between the cylinders. But if the surface area of
the vessel containing the liquid is suciently larger than the area between the
cylinders, the dierence layer will be thin and its energy shifts negligible.

Problem 5.1
Starting with the dierential expression
dB =

0 I 
x x
dl
4
|x x |3

for the magnetic induction at the point P with coordinate x produced by an increment of current I dl at x , show explicitly that for a closed loop carrying a current
I the magnetic induction at P is
B=

0 I

where is the solid angle subtended by the loop at the point P . This corresponds
to a magnetic scalar potential, M = 0 I/4. The sign convention for the
solid angle is that is positive if the point P views the inner side of the surface
spanning the loop, that is, if a unit normal n to the surface is dened by the
direction of current ow via the right-hand rule, is positive if n points away from
the point P , and negative otherwise. This is the same convention as in Section 1.6
for the electric dipole layer.
I like to change the notation slightly: the observation point is r1 , the coordinate of a point on the current loop is r2 , and the displacement vector (pointing
to the observation point) is r12 = r1 r2 .
The solid angle subtended by the current loop at r1 is given by a surface
integral over the loop:

cos dA
=
2
r12
S
1

Homer Reids Solutions to Jackson Problems: Chapter 5

current density is cylindrically symmetric, there is no vector potential in the


or z directions. In the direction we have
A = Ax sin + Ay cos = Ay

Jy (x )
0
dx
=
4
|x x |

J (x ) cos  
0
dx
=
4
|x x |


0
J (x )ei
Re
dx
=
4
|x x |







0
 i 2
Re
J (x )e
=
eim( ) cos[k(z z  )]Im (k< )Km (k> ) dk dx
4
m= 0

Solutions to Problems in Jackson,


Classical Electrodynamics, Third Edition
Homer Reid
February 11, 2001

Chapter 5: Problems 10-18

where we substituted in Jacksons equation (3.148). Rearranging the order of


integration and remembering that = 0, we have

Problem 5.10

A =

A circular current loop of radius a carrying a current I lies in the x y plane with
its center at the origin.
(a) Show that the only nonvanishing component of the vector potential is

0 Ia
A (, z) =
dk cos kz I1 (k< )K1 (k> )

A =

(b) Show that an alternative expression for A is



0 Ia
A (, z) =
dkek|z| J1 (ka)J1 (k).
2
0

Ia0


0

cos kz I1 (k< )K1 (k> ) dk.

(b) The procedure for obtaining this expression is identical to the one I just
went through, but with the expression from Problem 3.16(b) used for the Greens
function instead of equation (3.148).

(c) Write down integral expressions for the components of magnetic induction,
using the expressions of parts a and b. Evaluate explicitly the components of
B on the z axis by performing the necessary integrations.
(a) Translating Jacksons equation (5.33) into cylindrical coordinates, we have
(1)

Following Jackson, we take the observation point x on the x axis, so its coordinates are (, = 0, z). Since there is no current in the z direction, and since the
1

If m = 1, the integral yields 2; otherwise it vanishes. Thus



  
0
A =
J (r , z  ) cos[k(z z  )]I1 (k< )K1 (k> ) dz  dr dk
0
0

Substituting (1), we have

where < (> ) is the smaller (larger) of a and .

J = I(z)( a)


 



0
J (x )ei(1m) cos[k(z z  )]Im (k< )Km (k> )dx dk
Re
2 2
m= 0

(c) Lets suppose that the observation point is in the interior region of the
current loop, so < = , > = a. Then
A
B = [ A] =
z

Ia0
=
k sin kz I1 (k)K1 (ka) dk

0
1
A
Bz = [ A]z = A +




Ia0
I1 (k)
=
+ kI1 (k) K1 (ka) dk
cos kz

Homer Reids Solutions to Jackson Problems: Chapter 5

Homer Reids Solutions to Jackson Problems: Chapter 5

As = 0, I1 () 0, I1 ()/ 1/2, and I1 () 1/2, so

z1 = z

B ( = 0) = 0

Ia0
Bz ( = 0) =
k cos kzK1 (ka) dk

0

Ia0
=
sin kzK1 (ka)dk
z 0

z

The integral may be done by parts:






 1

z
sin kzK1 (kz) dk =  sin kzK0 (ka) +
cos kzK0 (ka) dk
a
a 0
0
0

z
I0
2 z (z 2 + a2 )1/2
2
a
I0
=
.
2 (z 2 + a2 )3/2

Bz ( = 0) =

Problem 5.11
A circular loop of wire carrying a current I is located with its center at the origin
of coordinates and the normal to its plane having spherical angles 0 , 0 . There is
an applied magnetic eld, Bx = B0 (1 + y) and By = B0 (1 + x).
(a) Calculate the force acting on the loop without making any approximations.
Compare your result with the approximate result (5.69). Comment.
(b) Calculate the torque in lowest order. Can you deduce anything about the higher
order contributions? Do they vanish for the circular loop? What about for
other shapes?
(a) Basically were dealing with two dierent reference frames here. In the lab
frame, R, the magnetic eld exists only in the xy plane, and the normal to the
current loop has angles 0 , 0 . We dene the rotated frame R by aligning
the z  axis with the normal to the current loop, so that in R the current loop
exists only in the x y  plane, but the magnetic eld now has a z  component.
The force on the current loop is

F = (J B)dV.
(2)

Homer Reids Solutions to Jackson Problems: Chapter 5


i
cos 0 cos 0
j = sin 0

sin 0 cos 0
k

cos 0 sin 0
cos 0
sin 0 sin 0

i
sin 0
j .
0

cos 0
k

y  = y1

x1

x1
x
R1 R

R R1

K0 is nite at zero but sin vanishes there, and sin is nite at innity but K0
vanishes there, so the rst term vanishes. The integral in the second term is
Jacksons equation (3.150). Plugging it in to the above,

frames:

z1
0

y1
x

(6)

Figure 1: Successive coordinate transformations in Problem 5.11.

The components of J are easy to express in R , but more complicated in R; the


opposite is true for B. There are two ways to do the problem: we can work out
the components of J in R and do the integral in R, or we can work out the
components of B in R and do the integral in R , in which case we would have
to transform the components of the force back to R to get the answer we desire.
I think the former approach is easier.
To derive the transformation matrix relating the coordinates of a point in R
and R , I imagined that the transformation arose from two separate transformations, as depicted in gure (??). The rst transformation is a rotation through
0 around the z axis, which takes us from R to an intermediate frame R1 . Then
we rotate through 0 around the y1 axis, which takes us to R . Evidently, the
coordinates of a point in the various frames are related by

sin 0 0
x1
cos 0
x
y1 = sin 0 cos 0 0 y
(3)
0
0
1
z
z1


x
cos 0 0 sin 0
x1
y = 0
y1
1
0
(4)
sin 0 0 cos 0
z
z1
Multiplying matrices,

cos 0 cos 0
x
y  = sin 0
sin 0 cos 0
z

cos 0 sin 0
cos 0
sin 0 sin 0

x
sin 0
y .
0
z
cos 0

This matrix also gives us the transformation between unit vectors in the two

Homer Reids Solutions to Jackson Problems: Chapter 5

so

To do the integral in (2) its convenient to parameterize a point on the


current loop by an angle  reckoned from the x axis in R . If the loop radius
is a, then the coordinates of a point on the loop are x = a cos  , y  = a sin  ,
and the current density/volume element product is

in exact agreement with the result we calculated so laboriously above.

+ (sin  sin 0 + cos  cos 0 )j + (sin  sin 0 )k

We also need the components of the B eld at a point on the current loop:
B( ) = B0 [1 + y( )]i + B0 [1 + x( )]

To compare this with the rst-order approximate result, note that the magnetic
moment has magnitude a2 I and is oriented along the z  axis:


 = a2 I sin 0 cos 0i + sin 0 sin 0j + cos 0 k

m = a2 I k

We will also the inverse transformation, i.e. the expressions for coordinates in
R in terms of coordinates in R :



cos 0 cos 0 sin 0 sin 0 cos 0
x
x
y = cos 0 sin 0
cos 0
sin 0 sin 0 y  .
(7)
z
sin 0
0
cos 0
z

J dV = Id l = (Ia d )
= Ia d [ sin i + cos j ]

= Ia d ( sin  cos 0 cos 0 cos  sin 0 )i

(5)





B m = B0 (1 + y)mx + B0 (1 + x)my


= B0 myi + mxj

= Ia2 B0 sin 0 sin 0i + sin 0 cos 0j)

Problem 5.12
Two concentric circular loops of radii a, b and currents I, I  , respectively (b < a),
have an angle between their planes. Show that the torque on one of the loops
is about the line of intersection of the two planes containing the loops and has the
magnitude
N=


2
2n

0 II  b2  (n + 1)
b
(n + 3/2)
1
P2n+1
(cos ).
2a
(2n + 1) (n + 2)(3/2)
a
n=0

= B0 [1 + a(cos  cos 0 sin 0 + sin  cos 0 )]i + B0 [1 + a(cos  cos 0 cos 0 sin  sin 0 )]j
The components of the cross product are
[J B]x dV = Jz By dV



= ( )Ia2 B0 d sin2  sin 0 sin 0

[J B]y dV = Jz Bx dV



= ( ) + Ia2 B0 d sin2  sin 0 cos 0

[J B]z dV = (Jx By Jy Bx ) dV
= ( ) + 0
where we only wrote out terms containing a factor of cos2  or sin2  , since only
these terms survive after the integral around the current loop (we grouped all
the remaining terms into ( )). In the surviving terms, cos2  and sin2  turn
into factors of after the integral around the loop. Then the force components
are
Fx = Ia2 B0 sin 0 sin 0
Fy = Ia2 B0 sin 0 cos 0
Fz = 0.

The torque on the smaller loop is





N = r Jb (r) Ba (r) dr
 





=
r Ba (r) Jb (r) r Jb (r) Ba (r) dr.
where Jb is the current density of the smaller loop and Ba is the magnetic eld
of the larger loop. But r Jb vanishes, because the current ows in a circle
around the originthere is no current owing toward or away from the origin.
Thus

N = rBr (r)Jb (r)dr
(8)
where Br is the radial component of the magnetic eld of the larger current
loop.
As in the last problem, its convenient to dene two reference frames for this
situation. Let R be the frame in which the smaller loop (radius b, current I)
lies in the xy plane, and R the frame in which the larger loop lies in the x y 
plane. We might as well take the line of intersection of the two planes to be the
y axis, so y = y  . Then the z  axis has spherical coordinates ( = , = 0) in

Homer Reids Solutions to Jackson Problems: Chapter 5

R, and for transforming back and forth between the two frames we may use the
transformation matrices we derived in the last problem, with 0 = , 0 = 0. If
we choose to evaluate the integral (8) in frame R, the current density is


Jb (r) = I(r b)( /2) sin i + cos j

Homer Reids Solutions to Jackson Problems: Chapter 5

To express the Legendre function in (11) with the argument (12), we may
make use of the addition theorem for associated Legendre polynomials:
Pl (cos  ) = Pl (cos cos + sin sin cos )
= Pl (cos )Pl (cos ) + 2

so the components of the torque are


 2
Nx = Ib2
Br (r = b, = /2, ) sin d
Ny = Ib2

Br (r = b, = /2, ) cos d

(9)

Plm (cos )Plm (cos ) cos m.

Of course, the smaller loop exists in the xy plane, so for all points on that loop
we have = /2, whence
Pl (cos  ) = Pl (0)Pl (cos ) + 2

(10)

l


Plm (0)Plm (cos ) cos m.

m=1

To do the integral in (8), we need an expression for the radial component Br


of the eld of the larger loop. Of course, we already have an expression for the
eld in R : in that frame the eld is just that of a circular current loop in the
x y  plane, Jacksons equation (5.48):
Br (r ,  ) =

l

m=1

0
2

2l+1
0 I  a  (1)l (2l + 1)!! r<
P
(cos  ).
2l+2 2l+1
2r
2l l!
r>
l=0

We are interested in evaluating this eld at points along the smaller current
loop, and for all such points r = b; then r< = b, r> = a and we have

2l

0 I   (1)l (2l + 1)!! b


P2l+1 (cos  ).
Br (r = b,  ) =
2a
2l l!
a

(11)

We may now write down an expression for the radial component of the magnetic
eld of the larger loop, evaluated at points on the smaller loop, in terms of the
angle that goes from 0 to 2 around that loop:

2l

0 I   (1)l (2l + 1)!! b


Br () =
P2l+1 (0)P2l+1 (cos )
2a
2l l!
a
l=0
!
2l+1

m
m
+2
P2l+1
(0)P2l+1
(cos ) cos m .
m=1

This looks ugly, but in fact when we plug it into the integrals (9) and (10)
the sin and cos terms beat against the cos m term, integrating to 0 in the
former case and m1 in the latter. The torque is

l=0

To transform this to frame R, we rst note that, since the origins of R and R
coincide, the unit vectors
r and
r coincide, so Br = Br . Next, (11) expresses
the eld in terms of cos  , the polar angle in frame R . How do we write this
in terms of the angles and in frame R? Well, note that
z
r
x sin + z cos
=
r
r sin cos sin + r cos cos
=
r
= sin sin cos + cos cos

cos  =

(12)

where in the second line we used the transformation matrix from Problem 5.11
to write down z  in terms of x and z. Equation (12) is telling us what our
coordinates in R are in terms of our coordinates in R; if a point has angular
coordinates , in R, then (12) tells us what angle  it has in R . (We could
also work out what the azimuthal angle  would be, but we dont need to,
because (11) doesnt depend on  .)

Homer Reids Solutions to Jackson Problems: Chapter 5

So my answer is
Ny =


2
2l

(l + 3/2)
b
0 II  b2 
1
(l + 1)2
P2l+1
(cos ).
a
(l + 2)(3/2)
a
l=0

Evidently Im o by a factor of 1/(l + 1)(2l + 1) under the sum, but I cant nd


where. Can anybody help?

Problem 5.13
A sphere of radius a carries a uniform surface-charge distribution . The sphere
is rotated about a diameter with constant angular velocity . Find the vector
potential and magnetic-ux density both inside and outside the sphere.

Nx = 0
Ny =

l=0

To nish we just need to rewrite the numerical factor under the sum:


(1)l (2l + 1)!! 1
(2l + 1)!!
(l + 3/2)
P2l+1 (0) =
2l l!
2l l!
(l + 1)(3/2)


(2l + 3 2)(2l + 3 4)(2l + 3 6) (5)(3)
(l + 3/2)
=
2l (l + 1)
(l + 1)(3/2)


(l + 3/2 1)(l + 3/2 2) (5/2)(3/2)
(l + 3/2)
=
(l + 1)
(l + 1)(3/2)

2
(l + 3/2)
=
(l + 1)(3/2)

2
(l + 3/2)
= (l + 1)2
(l + 2)(3/2)

Homer Reids Solutions to Jackson Problems: Chapter 5

Well take the cylinder axis as the z axis of our coordinate system, and well
take B0 along the x axis: B0 = B0i. To the extent that we ignore end eects,
we may imagine the elds to have no z dependence, so we eectively have a two
dimensional problem.
There are two distinct current distributions in this problem. The rst is
a current distribution Jfree giving rise to the uniform eld B0 far away from
the cylinder; this current distribution is only nonvanishing at points outside the
cylinder. The second is a current distribution Jbound = M existing only
within the cylinder. Since there is no free current within the cylinder or in its
inner region, the equations determining H in those regions are
B = (H) = 0,

H = Jfree = 0.

These imply that, within the cylinder and in its inner region, we may derive H
from a scalar potential: H = m , with m satisfying the Laplace equation.
In the external region, there is free current, so things are not so simple. To
proceed we may separate the H eld in the external region into two components: one that arises from the free current, and one that arises from the bound
currents within the cylinder. The former is just (1/0 )B0 and the second is
again derivable from a scalar potential satisfying the Laplace equation. So, in
the external region, H = (1/0 )B0 m .

10

So our task is to nd expressions for m in the three regions such that the
boundary conditions on B and H are satised at the borders of the regions.
Writing down the solutions of the 2-D Laplace equation in the three regions,
and excluding terms which blow up as 0 or , we have
"

n An cos n + Bn sin n
r<a
n=1 





"
n
n
m (, ) =
En cos n + Fn sin n
a<r<b
n=1 Cn cos n + Dn sin n +

" n G cos n + H sin n


r>b
n
n
n=1
Actually, we may argue on symmetry grounds that the sin terms must all
vanish: otherwise, the elds would take dierent values on the positive and
negative y axes, but there is nothing in the problem distinguishing these axes
from each other. With this simplication we may write down expressions for
the components of the H eld in the three regions:

Problem 5.14
A long, hollow, right circular cylinder of inner (outer) radius a (b), and of relative
permeability r , is placed in a region of initially uniform magnetic-ux density B0
at right angles to the eld. Find the ux density at all points in space, and sketch
the logarithm of the ratio of the magnitudes of B on the cylinder axis to B0 as a
function of log10 r for a2 /b2 = 0.5, 0.1. Neglect end eects.

2l

0 II  b2  (1)l (2l + 1)!! b


1
1
P2l+1
(0)P2l+1
(cos ).
a
2l l!
a

Hr =

m =
nAn n1 cos n,
r
n=1





m =
n Cn n1 En (n+1) cos n,
r
n=1




m = (1/0 )B0 cos +


nGn (n+1) cos n ,
(1/0 )B0r

r
n=1

H =

r<a
a<r<b
r < b.

m =
nAn n1 sin n,

n=1




=
n Cn n1 + En (n+1) sin n,

m
a<r<b

n=1




nGn (n+1) sin n ,


r < b.

(1/0 )B0 m = (1/0 )B0 sin +


n=1

The boundary conditions at r = b are that H and H be continuous,


where = 0 outside the cylinder and r 0 inside. With the above expressions
for the components of H, we have





1
B0 cos +
nGn b(n+1) cos n = r
n Cn bn1 En b(n+1) cos n
0
n=1
n=1

r<a





1
B0 sin +
nGn b(n+1) sin n =
n Cn bn1 + En b(n+1) sin n.
0
n=1
n=1

We may multiply both sides of these by cos n and sin n and integrate from

Homer Reids Solutions to Jackson Problems: Chapter 5

11

Homer Reids Solutions to Jackson Problems: Chapter 5

0 to 2 to nd

1
B0 + G1 b2 = C1 + E1 b2
0


Gn b(n+1) = Cn bn1 + En b(n+1) ,

n = 1

(a/b) = 0.5
(a/b) = 0.1

-0.5

(13)

-1

(14)
-1.5

(15)
n = 1

log10 r

1
B0 + G1 b2 = r C1 + r E1 b2
0


Gn b(n+1) = r Cn bn1 En b(n1) ,

12

(16)

Similarly, at r = a we obtain

-2
-2.5
-3

A1 = r C1 r E1 a2


An an1 = r Cn an1 En a(n+1) ,
A1 = C1 + E1 a2


An an1 = Cn an1 + En a(n+1) ,

(17)

-3.5

(18)

-4.5

n = 1

-4

(19)
For n = 1, the only solution turns out to be An = Cn = En = Gn = 0. For
n = 1, multiplying (15) by r and adding and subtracting with (13) yields
2r C1 = (r + 1)
2r E1 = (1 r )

B0
+ (r 1)G1 b2
0

B0 2
b + (r + 1)G1 .
0

The other coecients may be worked out from this one:


B0
4r b2
(r + 1)2 b2 (r 1)2 a2 0
B0
2(r + 1)b2
C1 =
(r + 1)2 b2 (r 1)2 a2 0
B0 2
2(r 1)b2
E1 =
a .
(r + 1)2 b2 (r 1)2 a2 0

(20)

A1 =

(21)

2r C1 = (r + 1)A1

(22)

2r E1 = (r 1)a2 A1 .

(23)

Equating (20) with (22), we nd


B0
(r 1)
+
G1 b2
0
(r + 1)

The H eld is
4r b2
B0
i,
r<a
(r + 1)2 b2 (r 1)2 a2 0
 a 2
 a 2

B0 
2b2

i 2(r 1)
=
(r + 1) + (r 1)
cos
r , a<r<b
(r + 1)2 b2 (r 1)2 a2 0
r
r


2

(b2 a2 )(2r 1)
B0
b
B0
,
i+
i
+
2
sin

r > b.
=

(r + 1)2 b2 (r 1)2 a2 0
r2

H=

while equating (21) with (23) yields


B0 b2
(r + 1)
A1 =
G1 a2
+
0 a2
(r 1)

The ratio r of the eld within the cylinder to the external eld is

and now equating these two equations gives



 a 2 
(2r 1)b2
B0
b2 .
G1 = 1
b
(r + 1)2 b2 (r 1)2 a2 0

This relationship is graphed in Figure

Homer Reids Solutions to Jackson Problems: Chapter 5

Figure 2: Damping of eld inside cylindrical cylinder of permeability r .

On the other hand, multiplying (18) by r and adding and subtracting with
(17) yields

A1 =

log10 r

n = 1.

r=

13

Problem 5.16

4r
2 .
(r + 1)2 (r 1)2 ab2

Homer Reids Solutions to Jackson Problems: Chapter 5

14

A circular loop of wire of radius a and negligible thickness carries a current I. The
loop is centered in a spherical cavity of radius b > a in a large block of soft iron.
Assume that the relative permeability of the iron is eectively innite and that of
the medium in the cavity, unity.

Since the iron lling the space r > b is assumed to have innite permeability,
the H eld (and hence the B eld, since B = H for r < b) must be strictly
radial at the boundary r = b. The An coecients are thus determined by the
requirement that (27) and (25) sum to zero at r = b:


0 Ia2  (1)n (2n + 1)!!  a 2n 1
An bn1 Pn1 (cos ) =
P2n+1 (cos ).
4b3 n=0 2n (n + 1)!
b
n=1

(a) In the approximation of b  a, show that the magnetic eld at the center of
the loop is augmented by a factor (1 + a3 /2b3 ) by the presence of the iron.

The orthogonality of the associated Legendre polynomials requires that each


term in the sum cancel individually, whence
A2n = 0

(b) What is the radius of the image current loop (carrying the same current)
that simulates the eect of the iron for r < b?
(a) There are two distinct current distributions in this problem: the free current
density J1 owing in the loop, and the bound current density J2 owing in the
iron. These give rise to two elds B1 and B2 , which must be summed at each
point in space to get the observed eld.
B1 is just the eld of a planar current loop, which Jackson has already
worked out for us in his section 5.5:


(1)n (2n + 1)!!  r 2n

0 I

P2n+1 (cos ), r < a

2a n=0
2n n!
a
B1r =
(24)



2 
n

2n
(1) (2n + 1)!! a

0 Ia

P2n+1 (cos ), r > a.


2r3
n n!
2
r
n=0

B1 =

0 I  (1)n (2n 1)!!  r 2n 1


P2n+1 (cos ), r < a
4a n=0
2n1 n!
a

0 Ia2  (1)n (2n + 1)!!  a 2n 1

P2n+1 (cos ), r > a.


4r3
2n (n + 1)!
r
n=0

(25)

n=0

B2 =


n=1


n=1

0 Ia2 (1)n (2n + 1)!!  a 2n


.
4b3
2n (n + 1)!
b2

Then the eld of the bound current in the iron is determined everywhere in the
region r < b:

0 Ia2  (1)n (2n + 1)(2n + 1)!!  ar 2n


P2n+1 (cos )
(28)
B2r =
4b3 n=0
2n (n + 1)!
b2
B2 =

0 Ia2  (1)n (2n + 1)!!  ar 2n 1


P2n+1 (cos ).
4b3 n=0 2n (n + 1)!
b2

(29)

As r 0, B2 0 and B2r 0 Ia2 /4b3 , while B1r 0 I/2a, so the total


eld at r = 0 is


0 Ia2
0 I
0 I
a3
Br (r = 0) = B1r (r = 0) + B2r (r = 0) =
+
=
1+ 3 .
2a
4b3
2a
2b
(b) The B2 eld may be attributed to an image current ring outside r = b if,
for suitable redenitions of I and a, the expressions (28) and (29) can be made
to look like the r < a versions of (24) and (25).

On the other hand, since J2 vanishes for r < b, the eld B2 to which it gives
rise has no divergence or curl in that region, which means that throughout the
region it may be derived from a scalar potential satisfying the Laplace equation:



B2 = m =
An rn Pn (cos )
B2r =

A2n+1 =

nAn rn1 Pn (cos )

(26)

An rn1 Pn1 (cos )

(27)

Problem 5.18
A circular loop of wire having a radius a and carrying a current I is located in
vacuum with its center a distance d away from a semi-innite slab of permeability
. Find the force acting on the loop when
(a) the plane of the loop is parallel to the face of the slab,
(b) the plane of the loop is perpendicular to the face of the slab.
(c) Determine the limiting form of your answer to parts a and b when d  a.
Can you obtain these limiting values in some simple and direct way?
(a) Well take the loop to be at z = +d, and the slab of permeability to
occupy the space z < 0, so that the boundary surface is z = 0.

Homer Reids Solutions to Jackson Problems: Chapter 5

15

In the region z < 0, there is no free current, so H = 0 everywhere; thus


H may be obtained from a scalar potential, H = m , and since H = 0
as well we have 2 m = 0. The azimuthally symmetric solution of the Laplace
equation in cylindrical coordinates that remains nite as z is

dk A(k)ekz J0 (k),
(30)
m (z < 0) =
0

and from this we obtain

H (z < 0) = m =
dk kA(k)ekz J0 (k)

0

kz
=
dk kA(k)e J1 (k)
0


Hz (z < 0) = m =
dk kA(k)ekz J0 (k).
z
0

(31)

Homer Reids Solutions to Jackson Problems: Chapter 5


to rewrite H1z as


Ia

dk ek(zd) J1 (ka) [J2 (k) J0 (k)] , z > d

4 0
H1z =


Ia

dk ek(dz) J1 (ka) [J2 (k) J0 (k)] , z < d.


4 0

Since the H2 eld arises entirely from bound currents, it may also be derived
from a scalar potential m satisfying the Laplace equation. The azimuthally
symmetric solution of the Laplace equation in cylindrical coordinates that remains nite for all and as z + is

m (z > 0) =
dk B(k)ekz J0 (k)
and the components of H2 are

dk kB(k)ekz J1 (k)
H2r (z > 0) =
 0
H2z (z > 0) =
dk kB(k)ekz J0 (k).

(36)
(37)

The required forms of the functions A(k) and B(k) are determined by the
boundary conditions on H at the medium boundary, z = 0:
H (z = 0 ) = H (z = 0+ )

so

H (z = 0 ) = 0 H (z = 0+ ).

Equating (32) with the sum of (??) and (??), we have

1
=
A
0 z


Ia
dk kek(zd) J1 (ka)J1 (k),
2 0


Ia

dk kek(dz) J1 (ka)J1 (k),


2 0

H1z =

(35)

(32)

On the other hand, for z > 0 we may decompose the H eld into two
components: one component H1 arising from the current loop, and a second
component H2 arising from the bound currents running in the slab. H1 is just
given by the curl of the vector potential we worked out in Problem 5.10:


Ia

0
dk ek(zd) J1 (ka)J1 (k), z > d

1
2

H1 =
A,
A = A ,
A =
0

0
Ia

0
dk ek(dz) J1 (ka)J1 (k), z < d.
2
0

H1

16

1 1
(A )
0
=

z>d

dk kA(k)J0 (k) =

0 Ia
2

dk kekd J1 (ka) (J2 (k) J0 (k)) +


0

dk kB(k)J0 (k)

z < d.
(33)





1
Ia

dk kek(zd) J1 (ka)
z>d
J1 (k) J0 (k)
2
k
0




Ia
1

J1 (k) J0 (k) ,
dk kek(dz) J1 (ka)
z < d.
2 0
k
(34)

In the last two equations we may use Jacksons identity (3.87),


1
1
J1 (k) = [J0 (k) + J2 (k)]
k
2

Homer Reids Solutions to Jackson Problems: Chapter 5

Solutions to Problems in Jackson,


Classical Electrodynamics, Third Edition
Homer Reid
April 20, 2001

Chapter 5: Problems 19-27

The tangential boundary condition at z = +L/2 is




m 
m 
=
z= L +
z= L
2
2





dk kA(k)ekL/2 J1 (k) =
dk k B(k)ekL/2 + C(k)ekL/2 J1 (k)
0

(1)

This must hold for all . Multiplying both sides by J1 (k  ), integrating from
= 0 to = , and using the identity

1
d Jn (k)Jn (k  ) = (k k  )
(2)
k
0
we obtain from (1) the relation
A(k) = B(k)ekL + C(k).

(3)

The perpendicular boundary condition at z = +L/2 is

Problem 5.19

Bz (z = L/2+) = Bz (L/2)

A magnetically hard material is in the shape of a right circular cylinder of length L


and radius a. The cylinder has a permanent magnetization M0 , uniform throughout
its volume and parallel to its axis.
(a) Determing the magnetic eld H and magnetic induction B at all points on the
axis of the cylinder, both inside and outside.
(b) Plot the ratios B/0 M0 and H/M0 at all points on the axis of the cylinder,
both inside and outside.
There is no free current in this problem, so H(, z) may be derived from a
scalar potential m (, z) satisfying the Laplace equation. Dividing space into
three regions

or


0 Hz (z = L/2+) = 0 Hz (z = L/2) + Mz (z = L/2)


m 
m 
=
+
M
()
z z= L +
z z= L
2
2




dk kA(k)ekL/2 J0 (k) =
dk k B(k)ekL/2 + C(k)ekL/2 J0 (k) + M ()

(4)

where
M () =

M1 ,
0,

<a
> a.



m =

dk A(k)ekz J0 (k),



dk B(k)ekz + C(k)ekz J0 (k),

dk D(k)ekz J0 (k),
0

Now we multiply both sides of (4) by J0 (k ) and integrate from = 0 to


= to obtain
z > L/2
L/2 < z < L/2


A(k) = B(k)ekL + C(k) + M1 ekL/2

J0 (k)d

= B(k)ekL + C(k) + (k)

z < L/2.
where we dened

(k) = M1 ekL/2
1


0

J0 (k)d =

(5)

aM1 kL/2
e
J1 (ka).
k

Homer Reids Solutions to Jackson Problems: Chapter 5

Homer Reids Solutions to Jackson Problems: Chapter 5

Problem 5.23

The solution of eqs. (3) and (5) is


B(k) =

1 kL
e
(k)
2

1
C(k) = A(k) (k).
2

(6)

From the boundary conditions at z = L/2 we may similarly obtain the relations
B(k) + C(k)ekL = D(k)
B(k) C(k)e

kL

= D(k) (k)

which may be solved to yield

A right circular cylinder of length L and radius a has a uniform lengthwise magnetization M .
(a) Show that, when it is placed with its at end against an innitely permeable
plane surface, it adheres with a force


K(k) E(k) K(k1 ) E(k1 )
F = 20 aLM 2

k
k1
where

1
B(k) = D(k) (k)
2

C(k) =

1 kL
(k).
e
2

2a
,
k=
4a2 + L2

(7)

a
k1 =
.
a2 + L 2

(b) Find the limiting form of the force if L  a.

Comparing (6) and (7) we nd


M1 a
kL
cosh
J1 (ka)
k
2
M1 a kL/2
B(k) = C(k) =
e
J1 (ka).
2k

A(k) = D(k) =

Then the components of the H eld are


kL kz

e
dk cosh
J1 (ka)J1 (k),
M1 a

2
0

kL/2
H = M1 a
dk e
cosh(kz)J1 (ka)J1 (k),

kL kz

e J1 (ka)J1 (k),
M1 a
dk cosh

2
0


kL kz
e
dk cosh
J1 (ka)J0 (k),
M1 a
2
 0
kL/2
Hz = M1 a
dk e
sinh(kz)J1 (ka)J0 (k),

kL kz

M1 a
e J1 (ka)J0 (k),
dk cosh

2
0

Well dene our coordinate system so that the z axis is the cylinder axis,
and well take the surface of the permeable medium at z = 0.
Our general strategy for this problem will be as follows. First, well nd
the magnetic eld H0 that exists in all space when the cylinder is pressed up
at against the innitely permeable medium. Then well calculate the shift dE
in the energy of the magnetic eld incurred by moving the cylinder up a small
distance dz o the surface of the medium. The force on the cylinder is then
readily calculated as F = dE/dz.
To calculate the energy shift incurred by moving the cylinder a distance dz
away from the permeable medium, we wont have to go through and completely
recalculate the elds and their energy in the new conguration. Instead, we
can use the following little trick. When we move the cylinder up a distance dz,
two things happen. First a gap of height dz opens between the surface and the
face of the cylinder, where previously there had been a xed magnetization M,
but now there is just free space. Second, between L and L + dz there is now a
xed magnetization M where previously there was none. Moving the cylinder of
xed M up a distance dz is thus formally equivalent to keeping the cylinder put
and instead introducing a cylinder of the opposite magnetization M between 0
and dz, while also introducing a cylinder of magnetization +M between L and
L + dz. The increase in eld energy in this latter case is fairly easily calculated
by taking the integral of 0 Mc
H0 over the regions in which the xed magnetization changes.
So the rst task is to nd the eld that exists when the cylinder is pressed
at against the surface. Since there are no free currents in the problem, we
may derive H from a scalar potential satisfying the Laplace equation. To begin we write down the general solutions of the Laplace equation in cylindrical
coordinates, observing rst that by symmetry we can only keep terms with no

z > L/2
L/2 < z < L/2
z < L/2

z > L/2
L/2 < z < L/2
z < L/2.

Homer Reids Solutions to Jackson Problems: Chapter 5


azimuthal angle dependence:

dk A(k)ekz J0 (k),



(m) =
dk [B(k)ekz + C(k)ekz ]J0 (k),

dk D(k)e+kz J0 (k),

z>L
0<z<L

(8)

z < 0.

The boundary conditions at z = 0 are that H and Bz be continuous. Assuming rst of all that the medium existing in the region below z = 0 has nite
permeability , the tangential boundary condition is


m 
m 
=
z=0
z=0+


dk k D(k)J1 (k) =
dk k [B(k) + C(k)]J1 (k).
(9)
0

Homer Reids Solutions to Jackson Problems: Chapter 5

Now taking , we see that, to keep the B and C coecients from blowing
up, we must have D 0. Then equation (??) tells us that B(k) = C(k), so
the middle entry in (8) may be rewritten:

dk (k) sinh(kz)J0 (k),
(0 < z < L).
m (z, ) =
0

The boundary conditions at z = L are




m 
m 
=
z=L+
z=L


m 
m 

=
+ M ()
z z=L+
z z=L
with M () dened as above. Working through the same procedure as above
yields the conditions

Multiplying (9) by J1 (k  ), integrating from 0 to , and using the identity


(2), we nd
D(k) = B(k) + C(k).
(10)

A(k)ekL = (k) sinh(kL)


A(k)ekL = (k) cosh(kL) + (k)
with (k) dened as above. The solution is

The normal boundary condition at z = 0 is of a mixed type. Below the line


we have simply Bz = Hz . Above the line we may write Bz = 0 [Hz + M ()],
where M () represents the xed magnetic polarization of the cylinder:
M () =

M,
0,

<a
> a.

(11)

The normal boundary condition at z = 0 is then







= 0
+ 0 M ()
m 
m 
z
z
z=0
z=0+



dk k D(k)J0 (k) =
dk k [B(k) C(k)]J0 (k) + M ()
0 0
0
Now multiplying by J0 (k  ), integrating from = 0 to , and using (2) yields


D(k) = B(k) + C(k)


M ()J0 (k) d.
(12)
0
0
Using (11), the integral on the RHS is
 a
Ma
M
J1 (ka) (k)
J0 (k) d =
k
0
where we dened a convenient shorthand. Then (12) is

D(k) = B(k) + C(k) (k).


0

(k) = (k)e+kL
A(k) = (k) sinh(kL).
Plugging these back into (8) and dierentiating, we nd for the z component of
the H eld


dk ekz cosh(kL)J0 (k)J1 (ka),


z>L
Ma
Hz (, z) =
0
(13)

M a
dk ekL cosh(kz)J0 (k)J1 (ka),
0 < z < L.
0

Now that we know the eld, we want to nd the change in energy density
incurred by putting into this eld a short cylinder (radius a, height dz) of
between z = 0 and z = dz, and another cylinder of the
magnetization M k
between z = L and z = L + dz. The
same size but with magnetization +M k
change in eld energy is just the integral of 0 M H over the volume in which
the magnetization density has changed:

dU = 20 M

dz


= 20 M dz

 L+dz  a
Hz (z, ) d dz + 20 M
Hz (z, ) d dz
L
0
 a

Hz (L, ) d
Hz (0, ) d
(14)


a

where in the last step we assumed that Hz remains essentially constant over
a distance dz in the z direction, and may thus be taken out of the integral.

CHAPTER 5. MAGNETOSTATICS

90

Chapter 5

y(t)

=-

E
E
E
wB cos(wt) + Bt + wB; z(t)

of radius

{3 whose

Magnetostatics
Sincev x B points upward, and that is also the direction of the force, q must be positive. To find R, in
I

terms of a and d, use the pythagorean theorem:

(R - d)2 + a2 = R2 =? R2

RV
{

y
-/3

(c)

m
~

= ~ .1

5.4

F = Ika2 z.1
Problem 5.5

= C2 cos(u;t) -

(a) K

CI sin(u;t) + C4.

(b) Assumingit starts from the origin, so C3 = -CI, C4 = -C2, we have i(O)

= 0 =? CI = 0 =? C3 =

Problem

0;

y(O)= 2~ =? C2u; + ~ = 2~ =? C2 = - 2~B = -C4; y(t) = - 2~B sin(u;t) + ~ t;


E
E
E.
E
z(t) = - 2u;B cos(u;t) + 2u;B' or y(t) = 2u;B [2u;t- sm(u;t)]; z(t) = 2u;B [1 - cos(u;t)]. Let (3 == E/2u;B.
Then y(t) = (3 [2u;t - sin(u;t)];
z(t) = (3 [1 - cos(u;t)]; (y - 2(3u;t) = -(3 sin(u;t), (z - (3) = -(3 cos(u;t) =?
(y- 2(3VJt)2
+ (z - (3)2= (32. This is a circle of radius (3 whose center moves to the right at constant speed:
Yo =2(3VJt; Zo = (3.

=B

=? CI

= -C3 = - u;B j
89

C2u; + B = B =? C2 = C4 = O.

= 21ra
1 ,

because the length-perpendicular-to-flow

(b) J = ~
s =>1 =

=? -ClUJ

Suppose f flows counterclockwise (if not, change the sign of the answer). The force on the left side (toward
the left) cancels the force on the right side (toward the right); the force on the top is laB = lak(a/2) =
lka2/2, (pointing upward), and the force on the bottom is laB = -lka2j2 (also upward). So the net force is

i(O)= 0 =? CI = 0, and hence also C3 = O.So y(t) = Et/ B; z(t) = 0.1 Does this make sense? The magnetic
==-qE, which exactly cancels the electric force; since there is no net force,
forceis q(v x B) ==-q(E/B)Bz
the particle moves in a straight line at constant speed. ..(

=B

r.vt), Zo = O.

5.3

Problem

= E/ Bj i(O) = O. Use these to determine CI, C2, C3, and C4.


y(O)= 0 =? CI + C3 = OJ y(O) = u;C2 + E/B = E/B =? C2 = OJ z(O) = 0 =? C2 + C4 = 0 =? C4 = 0;

(c) z(O) = y(O)

= wB sin(wt).
= {32. This is a circle

[1 + wt - cos(wt)] j z(t)

= CI cos(u;t) + C2 sin(u;t) + ~t + C3; z(t)

Problem

= z(O) = OJ y(O)

= (3(1 +

(b) FromEq. 5.3,mv = qBR =>;;, = FiR =

Problem 5.2
The general solution is (Eq. 5.6):

is at Yo

(a) From Eq. 5.2, F = q[E + (v x B)] = 0 =>E = vB => v

r"'",,~

p = qBR = IqB (a2 2d


+~)

center

= wB

z = {3sin(wt)j [y - {3(1+ wtW + z2

a2 + d2
2Rd + d2 + a2 = R2 =?R =
.
2d

The cyclotron formula then giyes

(a) y(O)

sm(wt). y(t)

4~A

Problem 5.1

y(t)

E.

= wB

Let /3 ==EjwB; then [y - /3(1 + wi)] = -(3cos(wt),

5.6

(a) v = r.vr,so K
Problem 5.7
I

=~

says V . (xJ)

J da

=a

= f7r.vr.1

!s ds de/>= 21ra

(b) v

= r.vrsin8~

=>

ds

is the circumference.

= 21raa

=>

= 21ra

1 ;J

= pr.vrsin8~,where
I

21ras
1 .1

p ==Qj(4j3)1rR3.

Iv pr dr =
(a;:)r dr = - (V . J)r dr (by the continuity equation). Now product rule #5
= x(V . J) + J . (Vx). But Vx = x, so V. (xJ) = x(V . J) + Jx. Thus Iv(V' J)xdr =

Iv V . (xJ) dr - Iv Jx dr. The first term is Is xJ . da (by the divergence theorem), and since J is entirely
insideV, it is zeroon the surfaceS. ThereforeIv(V' J)xdr = - Iv Jx dr, or, combining this with the y and
z components, Iv(V'
Problem 5.8

J)rdr

= - Iv J dr.

Or, referring back to the first line,

(a) Use Eq. 5.35, wIth


B=
. z = R,82 = -81 = 45, and four sIdes:
.
(b) z = R, 82= -81 = ;, and n sides:B = ~~~ sin(1r/n).

ii= /

~.
I~oll

J dr.

Qed

91
92

.
n/-LoI 7r
(c) For small B, sm B ~ B. So as n -t 00, B -t 2;R, ;: = I/-LOII
2R (same as Eq. 5.38, wIth
. z = 0).
Problem 5.9

CHAPTER 5. MAGNETOSTATICS

()

(a) The straight segments produce no field at P. The two quarter-circles give B

(~- i) I(out).

Problem

,\2 2

,\2

Problem 5.10

.
/-LaI
(a) The forces on the two sIdes cancel. At the bottom, B = 27rS => F
.

/-LoI2a

(_2 ) Ia = _2

/-LoI2a

/-LOI

7rS

7rS

(up). At the

/-LoI2a2

(a)

(b) The force on the bottom is the same as before, /-LoI2/27r


(up). On the left side, B = /-LoIz;
27rY

/-L 12

Fy

= - 2y37r
rq In (

s/J3 + a/2

= ks;

(b) J

1=

s/ J33

/-La12

= -rq2y37r In ( 1 + _2S )

12

is /-L;7r ['1 -

J3

force on the triangle

In

J3a
( 1 + ~ )] .

Jda =

211'ka3

ks(211's)ds= - 3

s3

= 13, for s < a; Ienc= I, for s > a. So B

Problem

Pol
{ -211'scjJ,
A

.""

B. dl

= Bl = polenc = polzJ

so IB =

:\ 600 :

V3

=>

-y

= -PoJzy

-poJa~,

for z > +a;

+I'oJay,

1m z

Problem

Jda =

ks(211's)ds=

< a;

> a. }

for s

(-a < z < a). If z > a,Ienc = polaJ,

> -a.

-1

~y

z{~ampedan

loop

5.15

The field inside a solenoid is ponI, and outside it is zero. The outer solenoid's field points to the left (-2),
whereas the inner one points to the right (+z).

Problem

~z

~~B
and
3/2 =~.
(a2 + z2)
a

Foran infinite solenoid, B2= 0, BI= 7r,so (cosB2- cosBd = 1 - (-1) = 2, and B =

So: (i) B
I

= poI(n1 -

n2)

z,

= -poln2

(ii) B
I

z, (iii) B
I

= 0.1

5.16

From Ex. 5.8, the top plate produces a field poK/2 (aiming out of the page, for points above it, and into
the page, for points below). The bottom plate produces a field poK/2 (aiming into the page, for points above
it, and out of the page, for points below). Above and below both plates the two fields cancel; between the plates
they add up to poK, pointing in.

dz

a2 sin3 B
.
(h - /-LonI
- /-LonI)~ sm
B - COSB1) .
B -- /-LonI
BdB-- /-LonI
cos BI - (COS2
( -a dB) - -2
a3 sin2 B
2
2
01
2

Problem 5.11
Use Eq. 5.38 for a ring of width dz, with I -t nI dz:
a2
B = /-LonI
3/2 dz. But z = acotB,
2
(a2 + Z2)

a
so dz = -~dB,
sm B
So

direction for z > 0, and in the +y di.rection for z < O.

_ J

Ienc =

5.14

By the right-hand-rule, the field points in the


At z = 0, B = O. Use the amperian loop shown:

31

211'a

PoI S2
---;!
211'a cjJ, for s

. The force on the right sIde IS the same, so the net

=> k =~.
A

211'ks 3

- dx. Here y = J3x, so


Y
.
..

27r 1s/V3

J3a

Puttmg m the numbers,

0,
for s < a;
_poI:i..
{ 21I'S'f', or s > a. }

= B 211's= polenc=> B =

B . dl

dF = I(dl x B) = I(dx x + dyy + dz z) x (~;~ z) = ~~:( -dx y + dy x). But the x component cancels the
correspondingterm from the right side, and Fy = -~

..

= v ~.EOPO

or v

v = \/(8.85 x 10-12)(411'x
1
10-7) = 13.00 X 108m/s.1 This is precisely the speed of light(!), so in fact you could
never get the wires going fast enough; the electric force always dominates.
G
Problem 5.13

top, B = 27r( S + a) => F = 27r( S + a ) (down). The net force IS 27rS( S + a) (up).

(s/V3+a/2) 1

repulsion per unit length on the other wire:

= -,
EO

Ie = -211'EOd
-. They balance when pov2

SoB =I~~ (1 + ~) I (into the page).

/-La12

= 21I'EO
1 ~. Electric
S

Electric field of one wire (Eq. 2.9): E

(b) The two half-lines are the same as one infinite line: :;~j the half-circle contributes ~~'.

/-LaI

5.12

Magnetic attraction per unit length (Eqs. 5.37 and 5.13): 1m = Po - v .


211' d

(a) B = poO'V (in) betweem the plates, I B = 0 elsewhere.


(b) The Lorentz force law says F = J(K x-B) da, so the force per unit area is f
I

I /-LonI.I../

to the right, and B (the field of the lower plate) is poO'v/2, into the page. So

=K

x B. Here K = (TV,

1m= PO0'2V2/2

(up).!

93
94

CHAPTER 5. MAGNETOSTATICS

(c) The electric field of the lower plate is O'/2Eo; the electric force per unit area on the upper plate is
I

Ie = O'2/2Eo (down).

They balance if POV2 =

Problem 5.17
We might as well orient the axes so the field point r lies on the y axis: r = (0,y, 0). Consider a source point
at (x',y',z') on loop #1:

dl'

~ = -x' x + (y - y') y - z' z; dl' = dx' X+ dy' y;


z
X
Y
0 = (-z' dy') X + (z' dx') y + [(y - y') dx' + x' dy'] z.
X ~ = dx'
dy'
-x' (y - y') -z'
Pol dl' x ~
Pol (-z' dy') X + (z' dx') y + [(y- y') dx' + x' dy'] Z
dB1=--=471"
1-3
471"
[(x')2+ (y - y')2 + (z')2]3/2
.

Now consider the symmetrically placed source element on


loop #2, at (x', y' , - z'). Since z' changes sign, while everything else is the same, the x and y components from dB1 and
dB2 cancel, leaving only a z component.
qed
With this, Ampere's law yields immediately:
B

(d) E

I/Eo,or v = 1/..,ftOiIO = c (the speed of light), as in Prob. 5.12.

e
N
M
d

atoms. moles. grams


mole gram volume

c'

p= (1.6x 10-19)(6.0 x 1023)(~.~)

=
=

(~ )
M

1.6 X

10-19

= !3oJm,where Jm is the magnetic current density (representing the,


I

IF = q, IE + (v x B)] + qm [B - ~(v x E)] .I'n this fo'm the magnetic aoalog to Coulomb',

1/<01'0'

f I Z dz = /-LoIZ t2

A = /-Lo
411"

Iz.

= -z

/-LolA
411"

C,

[I

V x A

(Z + v2Z

+ 82

)]! Zl

411"

= -

/-Lol l

Z2+V(Z2)2+S2

411"

( ~)

411"
..

82

V(Z2)2

or, sInce smlh =


/-LOI(sin()2
4
11"8

82

Zl

v(zd2 + 82

sin()l)

+ 82

V(Z2)2

- Zl-

+ 82
Zl

and sm()2=

;p I (as in Eq. 5.35).

+ 82 ]

+ 82

1
Zl

+ 82

+1
]

;p

+ v(zd2

+82

v(zd2

+ 82 V(Zl)2 + 82]

/-LoI

+ 82]

V(Z2)2 + 82

;p

;p

Z2

- 411"8[ V(Z2)2 + 82

Z2

~Z

'--""'"

z? - [(zd2 + 82] v(zd2

V(Zl)2

z
A

- 1-

Z2

[ V(Z2)2 +

V(Zl)2

[ Z2 +

Z2 - V(Z2)2 +82
1
411" [ (Z2)2 - [(Z2)2 + 82] V(Z2)2

- /-LoI8 -

[ Zl

_/-LoI8

Z2

= - oA;p = _/-LoI

or about 33 cm/hr. This

dz

411" JZ1 yz2 + s2

08

'7 64gm/mole,
- 9.0gm/cm3.

where

X 10-3

6.0 X io23 mole,

1 = 71"(2.5x 10-3)(1.4
1
(b) J =
pv =>v = 7I"S2p
x 104) = 9.1 X 10-3 cm/s,
is astonishingly small-literally slower than a snail's pace.
1112
Po
10-7) = 2 X 10-7 N/cm.
(c) From Eq. 5;37, 1m = 271" d
= (471" X271"

( )

C2 fm,
V2

(d), where

= 11.4 X 104 C/cm3.1

1~,
7I"S2 =

law reads F = 411"


0.0qmlr ;m2 f, so to determine 0.0we would first introduce (arbitrarily) a unit of magnetic charge,
then measure the force between unit charges at a given separation. [For further details, and an explanation of
the minus sign in the force law, see Prob. 7.35.]
Problem 5.22

= (e)(N)

charge of electron
Avogadro's number
atomic mass of copper
density of copper

ltI2

211"

= 12 X 1018 N/cm.1

/-Lo).The curl of E becomes V x E

= volume
charge = charge.
atom

/-Lo

v2

flow of magnetic charge), and!3o is another constant. Presumably magnetic charge is conserved, so Pm and Jm
satisfy a continuity equation: V. Jm = -oPm/ot.
As for the Lorentz force law, one might guess something of the form qm[B + (v x E)] (where qm is the
magnetic charge). But this is dimensionally impossible, since E has the same units as vB. Evidently we
need to divide (v x E) by something with the dimensions of velocity-squared. The natural candidate is

It doesn't matter. According to Theorem 2, in Sect. 1.6.2, J J . da is independent of surface, for any given
boundary line, provided that J is divergenceless, which it is, for steady currents (Eq. 5.31).
Problem 5.19
P

C2

Problem 5.21
~
At this stage I'd expect no changes in Gauss's law or Ampere's law. T4e divergence of B would take the
form V . B = o.OPm,where Pm is the density of magnetic charge, and 0.0 is some constant (analogous to (0

(the same as for a circular solenoid-Ex. 5.9).


For the toroid, N/271"s= n (the number of turns per unit
length), so Eq. 5.58 yields B = pon1 inside, and zero outside,
consistent with the solenoid. [Note: N/271"s = n applies only
if the toroid is large in circumference, so that s is essentially
constant over the cross-section.]
Problem 5.18

(a)

d
X 1010

c2

Ampere's law says V x B = /-LoJ. Together with the continuity equation (5.29) this gives V . (V x B) =
. J = -/-Loop/ot, which is inconsistent with div(curl)=O unless p is constant (magnetostatics). The other
Maxwell equations are OK: V x E = 0 =? V . (V x E) = 0 (./), and as for the two divergenceequations, there
is no relevant vanishing second derivative (the other one is curl (grad), which doesn't involve the divergence).

108m/s.

fe
(1.1 x 1025)(2 x 10-7)
Problem
5.20

. and

ponI Z, inside the solenoid;


outside

V2 211"fO

f,

= 3.00 x

C == l/yfO/-LO

( ) = ~~ 3 (0 ) = ( ) ( ) =
Here i = ;2 = (9.'1
) = 11.1 x 1025.1

fe = ~
211"fO

ltI2

/-LoV

{ 0,

= 211"fOd
~~;

A1A2

Zl

v(zd2

+ 82]

(j;
'

95

Problem 5.23

A",=k~B=VxA=--s as (sk)z=-z;s
Problem

fLo

fLo

(- )] cp=

-- a

5.24

-~V . (r

V. A
V xr

- J=-(VxB)=1

=0

=0

.B

and V

CHAPTER 5. MAGNETOSTATICS

V. (~) = -J.V' (~). But V'. (~) = ~(V"J)+J.V' (~), and V'.J = 0 in magnetostatics(Eq.5.31).So
J
.
, J
Po v , J
,
Po J d
d
h
V. -;; = - V.
-;; , and hence, by the Ivergence t eorem, V .A = - 47r
. -;; dr = - 47r -;;. a,
where the integral is now over the surface surrounding all the currents. But J = 0 on this surface, so V.A = 0./

()

zcp.

fLoS

= -~ [B. (V x r) - r. (V x B)] = 0, since V x B = 0 (B is uniform) and

x B)

= -2V

(Prob. 1.62). V x A

=0

(r. V)B

~-

96

= -2

x (r x B)

[(B. V)r - (r. V)B

+ r(V. B) - B(V. r)]. But

()

(b) V x A = ~:

J V 1x (~) dr' = ~; j

()

(since B is uniform), and V . r = ~~ + ~~ + ~: = 1 + 1 + 1 = 3. Finally,

= Bxx+Byy+Bz z = B.

(B.V)r = (Bx :x + By :y + Bz :z) (xx+yy+zz)

= -~(B-3B)

So VxA

~:

= B.

J (~) dr'.

ation with respect to r is concerned), and \72 (~)

coordinates,then, A = A(s) Z, so B = V x A = -

Problem

o1 ~ (the
aaAs ~ = fL27rS

aA = _fL

field of an infinite wire). Therefore

oI, and A(r) = _fL2o1


s
7r In(s/a) z (the constant a is arbitrary; you could use 1, but then the units
2~
aAz
= o.,( v x A = --aAz cp= - cp= B. ,(
look fishy). V. A =

_a

J.LoIs

aA

B = 21!'R2 cpo as = -

fLoI s

fLoI

21!' R2

= fLo 7r~2
...

7rS2

7rS2

= fL~:2

2
b ) z. Here b IS agalll arbItrary,

= - 41!'R2 (s -

= fLoJ

fLolenc

that

value; it works (say) for 0 ::;

Slllce A

-~~ In(R/a)

= - :;~2 (R2 - b2),which means that we must pick a and b such that
- fLoI ( 2
for s ~ R;
47rR2 S - R2) Z,
2In(R/b)= 1- (b/R)2. I'll use a = b = R. Then A =

must be continuous at R,

fLoI
- 21!'In(s/ R) Z,

for s

Problem 5.26

= Kx ~ B = xfLoK
2 Y (plus for z < 0, minus for z > 0).
A is parallel to K, and depends only on z, so A = A(z) x.

x
y
z
aA
a/ax a/ay a/az =8y=xfL~

B=VxA=1

JA= -Izi

A(z)

- U(a)] (by the gradient theorem), so U(b) f U(a).


I

qed

7r would do the job, in the sense that


U =- P2oIe/>
I

e/>

advances by 27r,this function does not return to its initial

at 27r it "jumps" back to zero.

r2

cos Br A

(3
(

R2

( 3 - 5 ) cosBr
(

Problem

)a

3r2
-powQ
1- 47rR [
5R2

=<i

R2

(a)

/ v. (~)

= ~;

;;.,

(:3 )

~ill do the job-or this plus any constant.

Problem 5.27

= -47r(53(1t)(Eq. 1.102).

- Powp .
R - B ;;.,
r dr + -rsm
'f'
r dr
3
r
r5
R2
r2
POWP .
Powp .
r
2
2
smB [ r2 5 + 2' (R - r) ] <p= ~rsmB
3 - 5 <p.
I
R2
r2
R2 r2
.
.
1
pow p
2.
V x A =--;-- - sm Br sm B - - r - - r sm B - - 2 { r smBaB [
3 .5
r or [
3
5
powpsinB
--'f'
3
r2

powp [

(a) V.A

y.

(once again, J is a constant, as far as differenti-

./

But when

< 27r,but

e/>

is not

Problem
5.29
Use Eq. 5.67, with R...,.+f and a ...,.+pdt:

R. }

;:::

. dl = -[U(b)

VU

= Pol
V(e/ = Pol ~~e/> Jy= B.
27r
27r s lie/>

-VU

except

= -poJ(r).

J(r') [-4m53(1t)] dr'

27rS Jy.
For an infinite straight wire, B = Pol

27rS

f B. dl = B 27rs =

5.28

f B . dl = -lb

Pol

- fLoI-

aS

(b) Here Ampere's law gives

= 47r
Po

= 0 (since J

(~) = J\72 (~)

But \72

\72

qed
Problem 5.25
.(a) A points in the same direction as I; and is a function only of s (the distance from the wire). In cylindrical

So \72 A

[~(V x J) - J x V (~)] dr'. But V x J


4
po JX4
,
= -2'IJ. (Eq. 1.101),so V x A = -47r ~ IJ. dr = B. ./

IJ.

a function of f), and V


(c) \72A =

()

2r2

- 5

(.
)
.
)

)]

) ] ()}

smB() ]

6r2
1- .
5R2

. But

= (4/3)7rR3'so

sm B() .
A

5.30

_a~z

= FY::}WZ(X,Y,Z)=-f;FY(X',Y,Z)dX'+Cl(y,z).

dr'. V. (~) = ~(V .J) +J. V (~). But the first term is zero, because J(r')

isa functionof the sourcecoordinates, not the field coordinates. And since~ = r - r', V (~) = - V' (~).

So

a~y = Fz::}Wy(x,y,z)=+f;Fz(x',y,z)dx'+C2(y,z).

These satisfy (ii) and (iii), for any C1 and C2; it remains to choose these functions so as to satisfy (i):

97

l
t

8Y

10

d '

X 8Fy(x',y,z)

8Cl -

x 8Fz(x',y,z)
8z
0

x+;::) uY

8Fx(x',y,z)
, 8Cl
8x'
dx+8y-8z

8~1 - 8~2 = Fx(O,y,z).


Wx =0;

NowJo

8Fy

ut;::) uX

+;::)uY

+;::)uZ

8x'

8Fz - 0

dx=Fxx,y,z-FxO,y,z,so

98

Wz= lY Fx(O,y',z)dy' -lx

8Wz - 8Wy

8Wy - 8Wx

8y

8y

Fy(x',y,z)dx'.

8z

~.

v'

in general.

= -+-+8x

8Wy

8Wz

8y

8z
x2

{X

(c) Wy

= 0+

dx ' +

x.8Fz(x',y,z)
0
8y

Y8Fx(0,y',z) d '
Y-

8z

.1

(~-zx)

X8Fy(x',y,z)
0
8z

d x '-t.I O,

{X

V x w~

8/8x
0

8/8y
x2/2

8/8z
(y2/2 - zx)

= a(~

,(

=yx+zy+xz=F.

(-

/loK(-y). 80 8Atove

8AYabove

8z

ponent 0 A parallel to K su ers a


Problem

5.33

)x + (

8AYbelow

8Axabove

8z

8z

8Axbelow

8z

.
d

..

lscontmmty -J.Lo

8) . .

is dt

211"
/UJ.

SO the

\"

current

in the ring is I = ~~ = aUJR2 sin 0 dO. The area of the ring


is 1I"(Rsin 0)2, so the magnetic moment of the ring is dm

RsinO

m = aUJ1I"R4fo" sin3 Ode = (4/3)aUJ1I"R\ or 1m = ~aUJR4 i.1


The dipole term in the multi pole expansion for A is there. .
~
- /Lo 411" R 4sin 0 ;., - /LoaUJR4sin 0;.,
h h
A
lore dip - 411"
3aUJ
7
'P 3
r2 'P, w IC IS
also the exact potential (Eq. 5.67); evidently a spinning sphere
produces a perfect dipole field, with no higher multipole contributions.
Problem 5.37
The field of one side is given by Eq. 5.35, with s --+

..JZ2+ {w/2)2 and sinO2=

sinOl =

/LoI

411"..Jz2 + {w2/4)..Jz2 + (w2/2)

component, multiply by sin 4>=

~~

) Y.
~

and

~~

are the same above

But E q 574 sa ys this e quals

. .

= 8A~~,OW,
and 8A~7ove 8A~:e,ow
= -J.LoK.Thusthe normalderivativeofthe comffi

For a ring, m = I1I"r2. Here I --+av dr = aUJrdr, so m = foR1I"r2aUJr


dr = 11I"aUJR4
/4.1
Problem 5.36
sin O)R dO.
The total charge on the shaded ring is dq = a {211"R

x r) = aVJRsin()(iJ,so the right side of Eq. 5.76 is

Because Aabove = Abelow at every point on the surface, it follows that


and below; any discontinuity is confined to the normal derivative.

=
B
- B
above
below

(2 cos0 f + sin

Problem 5.31
(a) At the surface of the solenoid, Babove = 0, Bbelow = J.LonIz = J.LoKz; ii = s; so K x ii = - K Z.
Evidently Eq. 5.74 holds. ,(
(b) In Eq. 5.67, both expressions reduce to (J.LoR2VJa/3)sin()(iJat the surface, so Eq. 5.75 is satisfied.
aA
2Sin () ;.,
J.LoR4VJa
2J.LoRv.Ja .
J.LoRv.Ja. () ;., 8 h I f .d
8A
_
() ;.,
f
ar R+ = 3
- ~r
'P R = 3
sm 'P;!:I ur R- =
3
sm 'P' 0 tee
t S1 e 0
I

I~; I :~2

{aUJR2sin 0 dO)1I"R2sin2 0, and the total dipole moment of the


shell is

y2

{y

Eq. 5.76 is -J.LoRVJasin() (iJ. Meanwhile K = av


- /l0(J1J.)
R sin ()(iJ,and the equation is satisfied.
Problem 5.32

(b) B ~

The time for one revolution

= Jo x'dx' = 2; Wz= Jo y' dy'- Jo Zdx' = 2" - zx.

Iw= ~Y+

5.34

(c) On the z axis, 0 = 0, r = z, f = i (for z > 0), so I B ~


i (for z < 0, 0 = 11",
f = -i, so the field
is the same, with Izl3 in place of Z3). The exact answer (Eq. 5.38) reduces (for z R) to B ~ /LoIR2/2IzI3,
so they agree.
Problem 5.35

)Z

But V. F = 0, so the x term is [Fx(O,y,Z)+ lx 8Fx~~:y,z) dX'] = Fx(O,y,z)+ Fx(x,y,z) - Fx(O,y,z),

soV x W = F. ,(
..., W 8Wx

Problem

(a)m=Ia=lhrR2i.1

d
(
)
)]
]
[
[ (
x ] x + 0 + Fy x, y, z y + Fz x, y, z - 0 z.
~

x - Jo

CHAPTER 5. MAGNETOSTATICS

,so

We may as well pick C2 = 0, Cl(y,z) = lY Fx(O,y',z)dy', and we'redone,with

Wy= lx Fz(x',y,z)dx';

Jo

(X 8Fx(x',y,z),

(
)
= Fxx,y,z.

8Wx - 8Wz y +
( 8y 8z )x'+ ({X8Fz(x',y,z)
8z
8x )
( 8x
'
r8Fy(x',y,z)

(b) V x W =
)
= [F( 0,y, z -

8C2

d ' - 8C2 - F (
x;::) uZ xx,y,z.

8Fx

, or, more compact y:

8Aabove

8n

8Abelow

8n

= -J.Lo .

(Sameidea as Prob. 3.33.) Write m = (m. f)f + (m. 0)0 = mcos()f - msin()O (Fig. 5.54). Then
= 2mcos()f+msin()O,
and Eq. 5.87 <=>Eq. 5.86. Qed
3(m.f)f - m = 3m cos() f - mcos()f+msin()O

sides, multiply by 4: IB

= /LoI

(w/2)

..Jz2 + w2/2

. To pick off the vertical

(w /2)
; for all four
..Jz2 + (w/2)2
W2

211"(z2 + w2/4)..Jz2 + w2/2

i. IFor

z w, B ~ /L;:~2 i. The field of a dipole 1m = IW2,I for


W/2
points on the z axis (Eq. 5.86, with r --+ z, f --+ i, 0 = 0) is
B-/Lomi
..(
- 211"
z3 .
Problem 5.38
The mobile charges do pull in toward the axis, but the resulting concentration of (negative) charge sets up
an electric field that repels away further accumulation. Equilibrium is reached when the electric repulsion on
a mobile charge q balances the magnetic attraction: F = q[E+ (v x B)] = 0 => E = -(v x B). Say the current

100

CHAPTER 5. MAGNETOSTATICS

99

= p_vz

is in the z direction: J

(where p- and v are both negative).

f B . dl =

E. da

B 27rS

= l1oJ7rS2 ~

= 110P2-vs

= E27rsl = .!..(p+
+ p_)7rs21 ~
fO

J;;

1 (p+ + p-)ss.
=_
2fO

Bin

~J.100"R(VZ=~J.100"R(V(cosBf-sinBO).FromEq.5.67,

Bout

V XA = V X

J.10R4(VO"
B
3r3 (2 cos r

J.1oR(VO"

3
.

=-

2~O(p+ + p-)ss

= ~Op_V2ss

[(vz) x (110P2-VS
J;)]

= p- (~:) .

= p-(fOI10V2)

~ p+ + p-

Problem 5.39
(a) If positive charges flow to the right, they are deflected down, and the bottom plate acquires a positive
charge.
I

(O"(VRsin B)

From Eq. 5.17, F = I J(dl x B). But B is constant, in this case, so it comes outside the integral: F

Problem 5.41

)[

(r x F) dt

r x q(v x B) dt

=q

= q [/ (r.

r x (dl x B)

B) dl-

= ma = qe( V

J.10qeqm

= -411"r
~

B)

SinB

( r )0

'

+ sinBf)sinB.

SoL = - 2: foR B 27rrdr = - 2: / B da. It followsthat L = - -!; <I>,where

(d) (i) Q . fjJ = Q(z

= r f,

<I>

cos

=-

2r

r2 dt

(r x v) . fjJ = O. But

= 0, so

fjJ

= r2 + r

(r2B)~.

qed

x v) . f - J.1o:~qm(f. f). But z. f = cosB, and (r x v) J.. r ~ (r x v) . f


.
.
J.1oqeqm
.

IS cons ant, so too IS Q . qe

= 0, so

J.1o:~qm
(f.O). But z.O = -sinB, f.O = 0, and (r x v).O = -r2 sinB~

2 . Bl
l
Q
sm 'f/ => 'f/ = mr
-:2

- Q sIn B = -mr
2.

= (-r2sinB~)0+

r rB r sinB~

= Q(z.O) = mer x v) .0-

'

(e) v2

= I rOO

A d
411" ' or Q = - 411"cos B. n smce

(1)) , so

x Bave) da. Here K = av, v = wR sin BJ;, da = R2 sin BdBd4J,and

Bave= !(Bin + Bout). From Eq. 5.68,

411"

- (f 'v)f - ~ + f 2(r .V) = O. v"

411" (f. fjJ). But z . fjJ= f.

= r2B= 0, so B is constant.

= Q(z. f) = mer
J.1oqeqm

(iii) Q.O
(from

411"r3

= J.1oqeqm ~
411" [ r

and v = dt
dl = r f + rB 0 + r sin B~ ~ (where dots denote differentiation with respect to time), so

(ii) Q . f

Problem 5.42

(r )

J.1oqeqm

B(r. dl)] .

= J B da

z.1

J.10qeqm

. fjJ) = mer x v) . fjJf

is the total flux.


In particular, if <I>= 0, then L = 0, and the charge emerges with zero angular momentum, which means it is
goingalong a radial line.
qed
I

= _~(O"(VR2)2

'

411" dt

Therefore (r x v) . ~

J (K

J.10

- sinBO)

(O"(VR2)2
211"Ci:4 B) C2, or F

( v x r;)

r xv

= -~o

sin3 BcosBdBdcj;

dt

But r i~ perpendicular to B, so r. B = 0, and r. dl = r. dr = !d(r. r) = !d(r2) = rdr = (lj27r)(27rrdr).

From Eq. 5.24, F =

= R) .

] = 6(O"(VR)-(4cosBO

fjJ x (4cosBf

The angular momentum acquired by the particle as it moves out from the center to the edge is

N dt

~~
r or

= J.1oqeqm~[r2v - (r. v)rJ - ~ + !...~(~)


411"
r3
r
r2 dt
r

.
+ sm B O) ( smce
r

cos B r

(2

Sin2

a = 411"
- ~mr (v x r ) .
1 dId
dv
dv
(b) BecauseaJ..v,a.v=O.
Buta,v=2dt(v,v)=2dt(v2)=vdt'
So dt =0.
qed
r
J.1oqeqm
d
- - -- 0 + J.1oqeqm[r X ( v x r J - J.1oqeqm -V --- r dr
( c) -dQ -- m (v x v ) + m (r x a ) -

I (Jdl) x B, and J dl = w, the vector displacement from the point at which the wire first enters the field to
thepoint where it leaves. Since wand B are perpendicular, F = I Bw, and F is perpendicular to w.

J.1oR(VO"

( r2 )

[ r smBoB

5.43

( a) F

Problem 5.40

Fz = -~o (O"(VR)2
R2
Problem

(b) qvB = qE ~ E = vB ::} V = Et = vEt, with the bottom at higher potential.


(c) If negative charges flow to the left, they are also deflected down, and the bottom plate acquires a negative
charge. The potential difference is still the same, but this time the top plate is at the higher potential.

~~ dt

(K x Baye)z = -~0(O"(VR)2sin2BcosB, so

~;,

~~

J.1oR4(VO"

Picking out the z component of 0 (namely, - sin B) and of f (namely, cosB), we have

Evidently p+ = -p- (1- ~:) =


or p- = _,2p+. In this naive model, the mobile negative charges fill a
smaller inner cylinder, leaving a shell of positive (stationary) charge at the outside. But since v c, the effect
is extremely small.

L=

(4 cosB r - SIn.BO)

J.10R(VO"

K X Baye

r2

+ sm BO) =

B aye =

~) =

J.10R4(VO"sinB

Q
J.1oqeqm
.
WIth k ==-m = - 41I"mcbs. B .

= r2'

. 2'

2 k2

=v

so r = v - r sin B4r

B2+ r sin Bcj;, but B = 0 and cj;= 2'


r

k2 sin 2 B
r2 .

101

()
defy
(f)
r(efy)

= 1-2 = V2 -

~2

/J

dr
2

(vr /k)

. 2 ()
sm

0 sm ()]'
= cos [(cjJ- AcjJ)'

Problem

(ksin()/r)2
(k2/r4)

/
=

Vr

[( k )

dcjJ* cjJ-

sm

Vr

cjJo

= --:--

2 ()

vr

= -k sm. () '

Problem 5.45
Let the source point be r'

or

= (s -

RcoscjJ) X - RsincjJy

...:

J.lo
41fKR

J.l41f io

KR

--

- zz.

(z2

{21f

(R R2

8 COS cjJ)

- 2R8COScjJ)3/2

82

sin </J

R2 d</J

(cas ()

J-LOl

47r

3;

cas </J

(sin () + sin </J) 0

cas </J)

dlx"=J-LOIR2z
~3

J-LoI

{21f

21f io

J.loK

[(R
-

roo

Problem

8V27rRZ

10

47r
1T

[I-cos((}+</J)]

1T

[2R2

dz

dz

+ 82 - 2RscoscjJ. Now i-oo (z2 + d2)3/2 =

2-

(R2
2

8)

210

"

(R-8COScjJ)

+ 82

- 2R8 COScjJ)

21f

5.46

d</J=

2R2 cos((}+ </J)]3/2

J-LoI

= I67rRz

J-LoIR2 Z
47r(2R2)3/2
0

(}+</J

21

{ n [tan

)]}I =

(~

1T

1T

d</J

VI

cos((} +

tan(~)
87rR n [ tan()
J-LoII

</J)

] z.

2z
d2vz2

00

+ d2

..

IB

. J-LoIR2

= ~

{[R2 + (d/2 + z)2]3/2

oz

2R

21f

tan

'I'

4
va2 - b2

-(d/2 + z)

a+ b

a+b'

(R2

J.loK

Bz = ~

IR2-82121f+21f] = 2

02B
21f

(-2 ) = va2-b2

. H ere

= R 2 + 82 ,

R2 - 82

( IR2-821 +1 ) .

the solenoid, 8 < R, so Bz = J.LoK


(1+1) = J.loK. Outside the solenoid, 8> R, so Bz = J.loK(-1+1)
2
2

= J.LonIz(inside),

oz2

3J-LoIR2

(d/2 - z)

{ [R2+ (d/2 + z)2]5/2+ [R2+ (d/2 - Z)2]5/2} .


{

-d/2

d/2

= 0 ./
.

[R2+ (d/2)2]5/2+ [R2+ (d/2)2]5/2}

-1

3J-LOIR2

-(d/2+z)(-5/2)2(d/2+z)
[R2+ (d/2 + z)2f/2
(d/2 - z)(-5/2)2(d/2 - Z)(-I)

{[R2+ (d/2 + z)2]5/2 +

-1
+ [R2+ (d/2 - z)2]5/2+
02B

oz2 z=O I

= O.

and O(outside) (as we found more easily using Ampere's law, in Ex. 5.9).
I

(b) Differentiating again:

]I 0
1f

va2-b2

J.loK

82)

1f

3/2

[R2+ (d/2 - z)2]

+ (-3/2)2(d/2-Z)(-I)
{ [R2+ (d/2 + Z)2]5/2 [R2+ (d/2 - z)2]5/2 }

=
oz z=O -

va2-b2tan(cjJ/2)

~n -1

va2 - b2tan(1f/2)

-1

oB

dcjJ.
]

1
0

2R8COScjJ)

= J-LoIR2 (-3/2)2(d/2+z)

oB

= d2'

dcjJ; (R-8COScjJ)=~[(R2_82)+(R2+82_2R8COScjJ)].

dA.

(R2 + 82 -

dcjJ
a + b cos cjJ-

va2-b2

d</J

0 V2sin[((} + </J)/2]

(a) From Eq. 5.38,

b= -2R8, so a2 - b2 = R4 + 2R282 + 84 - 4R282 = R4 - 2R282 + 84 = (R2 - 82)2; Ja2 - b2 = IR2 - d21,

= nI, so B

R2(sin </Jsin() + sin2 </J- cos(}cos</J+ COS2</J)d</Jz


R2 (1 + sin () sin </J - cas () cas </J)d</Jz = R2 [1 - cos((} + </J)]d</Jz.

(R-8COScjJ) { i-oo (Z2+d2)3/2 } dcjJ,

HereK

3J-LoIR2
J.loKR

(2"
defy
Jo a+bcoscjJ

Inside

=
=

dcjJ dz

roo

</Jd</Ji

-L

where d2 ==R2
-

dl X"

z
A

<P

Rsin</Jy, and

= R sin

X
y
Z
- sin cjJ
cos cjJ
0
(8 - R cos cjJ) (- R sin cjJ) (- z)
K [(- z cos cjJ)x + (- z sin cjJ)Y+ (R - 8 COScjJ)z] ;
1,2= z2 +R2 +82 - 2R8 COS
cjJ.The x and y components integrate
to zero (z integrand is odd, as in Prob. 5.17).

K x '"

Rcos</Ji

R [(cas () - cas </J)i + (sin () + sin </J)


y] and dl
Rcos </Jd</Jy Rd</J(sin </Ji + cas </Jy).

5.44

K(- sincjJx + coscjJy);'"

the field point be r = Rcos(}i + Rsin(}y; then"

Put the field point on the x axis, so r = (s, 0, 0). Then


J.lo (K x 4)
B = da- da = RdcjJdz' K = K
~
~'
,

Bz

- .

rv ( k )

()]
sm.
1
vr
.
sec-1 - '
sm ()
k sm () ; sec[(cjJ - cjJo)sm()]

where A ==- ~~~~n()


41fmv

= /

. dr
'defy

5. MAGNETOSTATICS

CHAPTER

102

dr

B(O)

:2

3J-LoIR2

{ [R2 + (d/2)2]5/2

3J-LoIR2

7/2

[R2+ (d/2)2]
J-LoIR2

(d2

[R2+ (d/2 - z)2f/2

3JloIR2
[R2 + (d/2)2f/2 } - [R2+ (d/2)2f/2

+ 2(5/2)2(d/2)22
- R2).

Zero if d
I

= R,

(-R2 - ~4 + 5~4 )

in which case

1
+
1
{ [R2+ (R/2)2]3/2 [R2+ (R/2)2]3/2 }

oIR2

Jl

(5R2/4)3/2

103
104

Problem 5.47
(a) The total charge on the shaded ring is dq = a(211"r)dr. The
time for one revolution is dt = 211"
/w. So the current in the ring
is 1 = ~;
.

= awr dr.
.

rIng (for pOInts on the axIs) IS dB

and the total field of the disk is


/1oaw

= ~

2"awr (r2 + z2)3/2 dr z,

3
r dr
(r2 + Z2)3/2 z.

10

Let u

=r

2
/1oaw
4 [

/1oaw
fR2. (u +udu
4 10
Z2)3/2 =

(R2 + 2Z2)

..jR2+ z2

= 2(R2 + z2) -

2Z

0R2

/1oaw
2

+ Z2 -

R2

But R2 + Z2 ~ R2 sin2 B + (Z2

- 2RzcosB. So

Z2

Bz

/1 pRw

~ 2

R2

(R2 + 2z2) - 2Z Z.
]
VR2 + Z2

/1opRw [II

~2 (12

13)

1 16R
60R3 z3

+ ZZ) + 12RZzZ) (Z + R)}

+ Z4 - R4 - 2Rzzz - z4 - ~RZzZ

RZzZ

Bz

Z Z

)=

15z3

J.1.oRpw3z
-(3z
[

5 Z
R +"2Z . /4

= Z J-I

Rz 2

{I

Rz

Rz
-Z

2R4
-15z3

Rz
-3z

But P

(R2/2) sin2 B
+ Z2 - 2RzcosB

(z

- RcosB) .
]

sin2B=1-u2.

udu

= O.

) - 2z

J.1.oQUJRZ A

= (4/3)l'OR3'so B =

101'Oz3Z.

5.48

J ;

(For simplicity I'll drop the prime on 4J.)

(y - Rsin 4J) z

-Rcos 4J

(R2/2)(1 - u2) - z + RU du

VR2 + Z2 - 2Rzu

= R J-I

/5

- 2z

B = J.1.o/ dl' Iz.. Iz. = -Rcos4Ji. + (y - Rsin4J)y + zz.


41'0
~

..jR2

{I

= 2Zj

+ R ) - --2 z + --2 15z3 R + -z


2

2Rz
-3z

2R5

J.1.0PW
15z3'

du

~z = Rz cosz 4J+ yZ - 2Ry sin 4J+ Rz sinz 4J+ zZ = Rz + yZ+ zZ - 2Ry sin 4J. The source coordinates (x', y', z')
satisfy x' = R cos4J~ dx' = - R sin 4Jd4Jj y' = R sin 4J ~ dy' = R cos4Jd4Jjz' = 0 ~ dz' = O. So dl' =
-Rsin4Jd4Ji. + Rcos4Jd4Jy.
.
i.
Y
Z
dl' X Iz.= I -Rsin4Jd4J
Rcos4Jd4J 0 1= (Rzcos4Jd4J)i. + (Rz sin4Jd4J)
y + (-Ry sin 4Jd4J+ Rz d4J)z.

+ 15] .

14

= -15RZz3 -"2R z - R

Bx

+ ZZ)Z + 8Rz(RZ + ZZ)+ 12RZzZ)(Z - R)

Problem

B:0~1I"=>u:1~-1j

+ z2 - 2Rzu -

[8(RZ + ZZ)Z - 8Rz(RZ

sodu=-sinBdBj

L1 [

+ 4(RZ+ zZ)2Rzu+ 3(2Rz)ZuZ)VRZ + ZZ- 2Rzu -1

/1opRw fl

- 60~3Z3 {[8(RZ

du

= J.1.oRpw
2z+

sinBdB VR2 + z2 - 2RzcosB -

10

2RzcosB + R2 COS2B) = R2

= -

ZZ - 2Rzu
1
60 R3 z 3 [8(RZ + ZZ)Z

.../RZ +

11"

Letu::cosB,
=

- Z]

R2/2

1-1

- 2z

..jR2+ Z2

..jR2 + Z2

UZ

= - 60R3z3 {z [16Rz(RZ + ZZ)]- R [16(RZ+ zz)z + 24RZzZ)}

, so du = 2r dr. Then
u + 2z2

( VU + z2 )]

R2

..jR2 + Z2

(b) Slice the sphere into slabs of thickness t, and use (a). Here
t = Id(RcosB)1 = RsinBdB;
a ~ pt = pRsinBdB; R ~ RsinB; z ~ z - RcosB. First
rewrite the term in square brackets:

From Eq. 5.38, the magnetic field of this


..
/10
r2
-

CHAPTER 5. MAGNETOSTATICS
1

= J.1.o/Rz1 Z".
41'0

= J.1.o1Rz

cos4J d4J

(RZ + yZ + ZZ - 2Ry sin 4J)3/2

41'0 Ry VRz

+ yZ + ZZ

Z".= 0,

2Ry sin 4JI 0

since sin 4J = 0 at both limits. The y and z components are elliptic integrals, and cannot be expressed in terms

of elementary functions.
fl

= L1
= -~

= - 3Rz

VR2 + Z2- 2Rzudu

3Rz [ (R2 + Z2 - 2Rz)3/2

12

(R2 + Z2 - 2Rzu)

(R2 + Z2 + 2Rz)3/2

3/2
1

1
-- 3& (z3- 3Z2R + 3zR2 - R3 - z3 - 3z2R - 3zR2 - R3)
1

1-1 ..jR2 + Z2-

2Rzu

B x-a,

-1

= -~3Rz [(z - R)3 - (z + R)3]


1

du=--vR2+Z2_2Rzu
Rz

-1

= -(3z2
~

B y- _J.1.o/Rz

d4J
10 Z". (RZ + yZ + sin4J
zZ - 2Ry

41'0

Problem

Rz

10

41'0

5.49
.
.
J.1.011 dli xi.
From the BIOt-Savart law, the field of loop #1 IS B = 4
~;
1'0 1
~

+ R2).

=--[(z-R)-(z+R)]=-.

. B z = J.1.o/R

3Z'

sin 4J) /

= lz1

1;

= J.1.0/Ilz
4

dlz x B

1'0

1 dlz
11;

X (~I X i).
~

(R - ysin4J) d4J

21r

(RZ + yZ + ZZ

3Z'
2Ry sin 4J) /

the force on loop #2 IS


.

= dlI(dlz .i) - i(dli . dlz),

Now dlz X (dli X i)

so

105

=-

J.lo

47rhI2

4
1.2(dl1 . dl2) -

{ff

f f
dl1

1.2

The first term is what we want. It remains to show that the second term is zero:

= (X2 -

Here Ampere's

x + (Y2-

yd Y+ (Z2- Zl) z, so V2(1/1,) = 00X2 [(X2- xd2 + (Y28


2
0
2
2
2
2 -1/2
+
)
X2
Xl
+
Y2
Y1
Y+ -0 [(X2 - Xl ) + (Y2 - Y1) +
) + ( Z2 - Zl ) ]
(
8~ [(
~
4
= - (x2-xd~ x- (Y2-Y1)~ y- (z2-zd~ z = -- -t = --.4 So -.d}z = - V2
It

Xl)

1.3

1-3

2 in Sect. 1.3.3).
Problem 5.50

1-3

1-3

1-2

qed

2 -1/2

(Z2 - zd ]

(Eq. 2.24) says \72V

V.

!
!

V(r')83(r -r')dr'

(~) = -V'. (~) = 47r!53(-t) = 47r!53(r -

= -~47r

()

v(r')V',

1.2

Conclusion: (ii) does not automatically yield V . A


Problem 5.52
(a) Exploit the analogy with the electrical case:

=~
47r

dr'

(d) B

= J.loI
(p, so
27rS

B x dl

27ra

27rb

= -r x B fOl>'d>'= I-~(r x B).I


J.loI
= -J.loI
cjJ; B(>.r)= cjJj A =
27rS
27r>.s
~

vector from the origin-in

(x~)=z, (zx(p)=-s.

J.lOI8 - J.loI 8 w

4 . [V'V(r') ] dr' 1-2

~
47r f

(~ - ~) 8

J.lolw
27r

=1=

27rs

cylindrical coordinates r

10

x cjJ)

v(r') 4 .da'
1.2

E(r') . 4

= S s + z z.

>.

So A

1.2

dr', as

/-Lo r13[3(m . f) f - m]
411"

p]

(Eq. 3.104)

~2SZ

~~.I

=F

sz).

O.

(Eq. 3.102).

= (411"/3)VJaR4
z (which we alsogot

/-LoVJaR4cos()

as well pick the constant to be zero, so U(r)

= -~/-LoaVJRrcos() for r < R.


= R): Uin(R) =
I

[Notice that U(r) is not continuous at the surface (r

atjJ

r 8()

8U
8r

-~/-LoaVJR2
cos() =FUout(R) =

=
-

=>

cos () r
~

(-- )
6r2

5R2

sm ()()
]

1 au
= - VU = - -au
r - - - () 8r
r 8()
~

U(r, (), tjJ) = U(r, ()).

6r2

( 411"R)(1- 5R2 )sm() => U(r,()) = - ( 411"R)(1- 5R2 ) rcos() + f(r).


- ( 411"R) (1-5R2 ) cos()=>U(r,())=( 411"R) (r-5R2 ) cos() + g()).
/-LoVJQ

6r2

J.LoVJQ

/-LoVJQ

3r2

/-LoVJQ

( 411"R) (1J.LoVJQ

f;

6r2

/-LoVJQ

5R2 rcos() + f(r) = -

( 411"R)(1-

r3

or
/-LoVJQ

( )
411"
R3

r3 cos()

+ f(r)

= g()).

r2

8U

-ljJ.

r sin () 8tjJ

Equating the two expressions:

V . A = -V. (r x L) = -[L . (V x r) - r . (V xL)] = r. (V x L), and


V x L = fOl>'[V x B(>.r)]d>'= f; >.2[VAx B(>.r)]d>'= J.lo >.2J(>.r)d>',so V . A = J.lor. fOl >.2J(>.r) d>', and

it vanishes in regions where J = 0 (which is why the examples in (c) and (d) were divergenceless). To construct
an explicit counterexample, we need the field at a point where J =1=
a-say,
inside a wire with uniform current.

3r2

= -/-LoVJQ
1- 411"
R [(
5R2

18U

27rs
J.loI
- 27rs s(s X cjJ)+ z(z x cjJ) , and

= - VV, withV = _411"100


1 pr ~ f

x cjJ). But r here is the


~

/-LoJz

(Eq. 5.87) = - VU, (Eq. 5.65).

SOIA=~(ZS-sz.1

f; >'B(>.r)d>',for short)

(s ) =

i/-LoaVJR2cos(). As I warned you on p. 236: if you insist on using magnetic scalar potentials, keep away from
places where there is current!]
(c)

The examples in (c) and (d) happen to be divergenceless, but this is not the case in general. For (letting
L ==

13[3(p . f) f

11"100
r

m Prob. 5.36). USIng the result of (a), then, U(r) =


3
r2
for r > R.
Inside the shell, the field is uniform (Eq. 5.38): B = ~/-LoaVJR
z, so U(r) = -~J:toaVJRz + constant. We may

O.

1
J.loI
>.- d>' = --(r

_
41

8U
J.loI
--(r

/-LoJs
x ljJ) = ~(zs

= 0.1

I =1=

(c) A

[s

= /-LoJ

+ ~(-S2)
8z

~~(S2Z)
8s

Evidently the prescription is p/o -t /-Lom: U(r) =


(b) Comparing Eqs. 5.67 and 5.85, the dipole moment of the shell is m

r'),

.
1
(Eq.1.59). But V'V(r') = -E(r'), and the surface mtegral --+0 at 00, so V(r) = - 47r
before.You can also checkthe result, by computing its gradient-but it's not easy.]
Problem 5.51
(a) For uniform B, f;(B x ell)= B x f; dl = B x r
A = -HB x f).
(b) B

/-LoJ

1.

/-LoJ

(2 ) '\sljJd'\ = -Ts(r

equation

/-LoJ

-r x Jo ,\

V.A

.
1
E(r') .4
sothe analogy ISP --+-foE, and hence V(r) = - 47r
1.2 dr'. qed
[Thereare many other ways to obtain this result. For example, using Eq. 1.100:

V(r) =

A =

f (- ) .dl2 = 0 (by Corollary

1.2

law gives B 211"s= /-LoIenc= /-LoJ1I"S2=> B = /-L~JS 4>, so

yd2 + (Z2- Zl)2r1/2 X

_-.!:..p. For dielectrics (with no free charge), Pb = -V. P


fO
1
(Eq. 4.12), and the resulting potential is V(r) = 47rfO P(r']1. .4 dr'. In general, P = fOV . E (Gauss's law),
Poisson's

CHAPTER 5. MAGNETOSTATICS

106

(dh'4)

5R2 rcos() + g()),

107
CHAPTER

108

But there is no way to write r3 cos() as the sum of a function of () and a function of r, so we're stuck. The
reason is that you can't have a scalar magnetic potential in a region where the current is nonzero.
Problem 5.53
/Lo J
(a) V . B = 0, V x B = /LoJ,and V . A = 0, V x A = B =>A = -Ij. dr', so
411"

! {(V x A3)' (V x A3) - A3' [V x (V x A3)]} dr = !

{(B3)'

(B3) - A3'

5. MAGNETOSTATICS

[V x B3]}

dr

!(B3)2dr

0), or else B

= f

V. A = 0, V x A = B, and V. W = 0 (we'll choose it so), V x W = A => W = -};


~ dr'.1
(b) W will be proportional to B and to two factors of r (since differentiating twice must recover B), so I'll
try something of the form W = ar(r . B) + .8r2B, and see if I can pick the constants a and .8 in such a way
that V . W = 0 and V x W = A.
I

8x

8y

(V

x A3)]

(in which

= f

. da

case

hence B1 = B2.

(A3

B3)

. da.

But

either

is

specified

= 0), at the surface. In either case f(A3

B3

(in

x B3)

which

case

.da

O.

A3

So

is

(B3)2 dr = 0, and

Qed

Problem 5.55

8z

V. W= a [(r. B)(V . r) + r. V(r. B)] +.8 (r (V. B) + B. V(r )] . Vr = 8x + 8y + 8z = 1 + 1 + 1 = 3;


V(r. B) = r x (V x B) + B x (V x r) + (r. V)B + (B. V)r; but B is constant,so all derivativesof B vanish,
andV x r = 0 (Prob. 1.62), so

From

"

Eq. 5.86, Btot = Bo z - J.L


1f:r (2cos()r+sin()6).
4om3o

B
.

This

= B0 Z .r

lore

= (B . V)r = (Bx :x + By:y + Bz :z) (x x + YY+ z z) = Bx x + By Y+ Bz z = Bi

V(r . B)

[A3

specified

J.Lorno

()

411"r3 cos

= (B0 -

by Bo = ~;~~, or

r = R, given

is zero, for all,(),when

There-

J.Lomo
()
) cos.
211"r3

1/3

= (x :x + Y:y + z :z) (x2 + y2 + z2) = 2x x + 2y Y+ 2z z = 2r.

So
V . W = a [3(r . B) + (r . B)] + .8 [0 + 2(r . B)] = 2(r . B)(2a + .8), which is zero if 2a + .8 = o.
V x W = a [(r. B)(V x r) - r x V(r. B)] +.8 (r2(V x B) - B x V(r2)] = a [0 - (r x B)] +.8 [0 - 2(B x f)]
V(r2)

= -(r

- 2.8) = -~(r

x B)(a

x B) (Prob. 5.24). So we want a - 2.8 = 1/2. Evidently a - 2(-2a)

R =

Evidently

1= (2~W)= ~:;

(a)

no

field lines cross

L
m = '2
Q MwR2
wR'

W to point parallel to the axis, and choosingW = 0 on the


axis:

r (/LonI ) lsds = /LonI821.


~2
(usmg Eq. 5.70 for A).

= 2M'
Q

(b) Because

a=1I"R2;m=

Iw= -~zl
1:'

I'Or s

> ,

(8< R).

r (/LonI
Id -= /LonIR21 /LonIR21 1 ( /R)
2 ) s
4
+
2
n 8
,

g is independent

= _/LonIR2
[1+ 2In(8/R)]
4

Problem

of R, the same

411"R3

cos ()

R2

sin() d() d4> =

27rR2Z
.

27rR2Z

- 2Rz'u

2 [2(R2 + (z')2)

Jo

- [3(~~)2z] {[R2
47r
""3Z

47r

Z -

""3r

47rR3

VR2

+ (Z')2 + Rz'] IR (

'

cas () sin ()

+ (z')2

VR2

+ (z' ) 2 -

Eave

(r

> R).

2Rz'u

( )

1
411"R(4/3)1I"R3

J-LoUJQ

811"
R4 R

-1

1r

qed

3/Lo 47r 3

b ThIS tIme r > R, so Bave = - (47r)2R3 3 R


from the source point to the center (~= ~r'). Thus

2Rz' - [R2 + (Z')2 - Rz'] VR2 + (Z')2

/Lo

J (J x

Bave

(r')3
= Been.

'

/Lo

T = 47r
qed

J~

dT

J x Ii.

(a)Problem5.51givesthe dipolemomentof a shell: m = 4; (1wR4Z. Let R -7 r, (1 -7 P dr, and integrate:

m = 47r
""3wpz Jo
{R r 4 dr = 47r
""3wPTR5 z.
A

But p = (4/3)7rR3'
Q

so m = SQwR
1
I

(4/3)1I"R3

z.
A

16

1r

1r

3J-LoUJQ1
811"
R

B) 0

6 R5

COS2B
2

- 3cos

R3

- "55R2

75 cos B

Ac5 (r)dr

5.60

J-LoUJQ

75

J-LoUJQ

= 2001l"R(20) = 1O1I"R(same

00

J-LO

J-Lo

2m

as (b)). ./

J dr

= 41I"rddPt
J-LO

- 2J-Lom

- 411"R3A - 411"R3 =} A -~.

IA = :; n=O
L rn~1 (r,)n Pn(cosB)J

(a) I dl -+ J dr, so
(b) Amon

dT, where ~ now goes

6r2

) cos B + (1 - 5R2 ) sm B r smOdr d(}dif>


) + (3 - "55R2 ) sin2 B] sin BdB
.
7.
1 (7 + 9 cos B) sm BdB
+ 75 sm B) sm BdB =

Problem 5.59
The issue (and the integral) is identical to the one in Prob. 3.42. The resolution (as before) is to regard
Eq. 5.87 as correct outside an infinitesimal sphere centered at the dipole. Inside this sphere the field is a
delta-function, Ac53(r), with A selected so as to make the average field consistent with Prob. 5.57:

Problem

) aT' = - 47rR3 (J xr') dT'. Now m = :2j(rxJ)

r'

10

3r2

[ 1 - 5R2
r3
3 R5

[( 3"

1r

2001l"R (-7cosB

Bave

Problem 5.58

!(

J-LoUJQ

+ 2Rz'}

J (J xr

coordinates so that rl liesonthez

(x,y,z)

3J-LoUJQ 3

'

3(z')2 Z = ""3 (r')3 r,

CHAPTER 5. MAGNETOSTATICS

d().

[R2 + (Z')2 - Rz'] (R + z')}

z'l-

f {! 11-

110

- 2Rz' cos()

< R) ,

3/Lo 47r
Fornowwe want r' < R, so Bave = - (47r)2R33
.;

=14.61 X 10-24 Am2.1

axis (see diagram). Then 11= VR2 + (ZI)2 - 2RzICOS(), while


da = R2 sin ()dOd4>r. By symmetry, the x and y components
must integrate to zero; since the z component of r is cos (), we
have

+ Rz'] VR2 + (z')2 -

/Lo 2m
(Eq. 5.91), so Bave = 47r R3'

411"R3 411"

(411"~2)2211" 0

+ 2Rz'u]

47r R3,

x 10-34)

4(9.11
x 10-31)

{f ;da} drl. Note that J depends on the


r/, not on the field point r. To do the surface

du

3(2Rz')2

3(Rz')2 { [R2 + (Z')2

to the entire sphere

obviously points in the z direction, so take the z component of r (cos B) and {J (- sin B):

27rR2Z

hence

J x

3J-L UJQ

+ (Z')2

-1 VR2

27rR2 Z

VR2 + (z')2 - 2Rz' cos()


Let u ==cas (), so du = - sin ()d().

integral, choose the

109

da =

ratio applies to all "donuts", and

2~.1

(3/4~1I"R3! B dr = 411"~3!(V x A) dr =
= - - 3 -J.Lo
-J dr I x da =

A x da

source point

j {(V x V) . (V x U) - U . [V x (V x V)]} dr = f [U x (V x V)] . da.

in the above identity:

1-

- (4:~oR3

Asalways,suppose we have two solutions, B1 (and Ad and B2 (and A2)' Define B3 = B2 - B1 (and
A3==A2 - Ad, so that V x A3 = B3 and V x B3 = V X B1 - V X B2 = /LoJ - /LoJ = O. Set U = V = A3

f~

5.57

(a) Bave =
3

--

x V). (V xU) - U. [V x (V x V)]:

V . [U x (V x V)] dr

x 10-19)(1.05

(1.60

4m

z 1(8> R).

Problem 5.54
Apply the divergence theorem to the function [U x (V x V)], noting (from the product rule) that

= (V

=~ =

2m 2

.,

d h

MwR2;L= MwR2Z.

V. [U x (V x V)]

= ~~

(c) m

'

1R

L=RMv=

(or any other figure of revolution):

R2

- Wl = /LonIR21

~:1I"R2Z= ~WR2Z.

= 2M)L,an t e gyromagnetlcratioIS LNJ

= 10 ~

-Wi

this sphere.

= 5a = 1/2,

= 1/10;.8 = -2a = -1/5. Conclusion: W = (r(r. B) - 2r2B] .1 (But this is certainly not unique.)
(c) V x W = A => J(V x W) . da = J A. da. Or fW. dl =
JA . da. Integrate around the amperian loop shown, taking
or 0:

( ~;;~ )

Problem 5.56

2J-LO

The added term IS "3mc5

(r).

dr.

(Prob. 5.7), where P is the total electric dipole moment. In magne-

41I"r
to statics, P is constant, so dp/dt = 0, and hence Amon = O. Qed
(c) m = Ia = ~I (r X ill) -+ m = ~ J(r X J) dr. ,Qed
Problem 5.61
For a dipole at the origin and a field point in the x z plane (if>= 0), we have
B

=
-

1I"rn;
41I"rn; (2 cos Br + sin B9) = 4J-LO
J-LO

[2 cos B(sin B x + cos B z) + sin B( cos B X - sin B z)]

n; [3 sin B cos ()x + (2 COS2B - sin2 B) z].


4 1I"r
J-LO

(b)

ave

/Lo 2m -

(c) A ~ /Lomsin()
47r

/Lo 2Qw A

- 47rR3 - 47r 5R Z.
r2

$=

(d) UseEq. 5.67, with R

/Lo QWR2 sin()


47r

r2

-7 r,(1 -7 par,

Here we have a stack of such dipoles, running from z =


-L/2 to z = +L/2.
Put the field point at s on the x

$.
and integrate:

A = /Lowpsin () $ {R r4 dr = /Low

r2

Jo

~
sin () R5 $ =
47rR3 r2 5

/Lo QwR2 sin ()

47r

r2

$.

This is identical to (c); evidently the field is pure dipole, for points outside the sphere.
(e) According to Frob. 5.29, the field is B =

/L:;~[(1- :~:

cas ()f

- (1 - :~2 ) sin() {j]. The average

axis.
The x components
cancel (because of symmetrically placed dipoles above and below z = 0), leaving B =
J-Lo
L/2 (3 COS2B - 1)
3
dz, where M is the dipole mo4 2M z

10

11"

= I1I"R2 = (CTvh)1I"R2 =
.
8
1
sin3 B
= 1I"CTUJR. Now smB = -,r so r = ~;8

ment per unit length: m


M

-m
h

/2

-8 cotB =} dz = sin
~ B dB. Therefore

"3"

=}

CTUJR1I"R2h

-/2

111

J1-

9m

sin3

()

J1-oaCI.JR3

228
8

But sin()m=
J82

+ (L/2)2'

9m

z(-cos()+cos())

rr/2

and cos() =
m

228

11r/2 (3cos2()-I)sin()d()

J1-oaCI.JR3

228

9m

J1- aCI.JR3

o (7raCI.JR3)zrr/2 (3cos2()-I)~~d()=
27r
8
sm ()

-(L/2)

J82 + (L/2)2'

so B =
I

J1-oaCI.JR3

COS()m(1-COS()m)Z=

228

COS()msm2()mZ.

J1- aCI.JR3 L
~

4[82 + (L/2)2]3/2 z.

Вам также может понравиться